You are on page 1of 24

CHAPTER

9 Environmental and Nutritional


Diseases

PBD9 Chapter 9 and PBD8 Chapter 9: Environmental and Nutritional Diseases

BP9 Chapter 7: Environmental and Nutritional Diseases

BP8 Chapter 8: Environmental Diseases

1 In an experiment, the effects of xenobiotic activation of 4 A 75-year-old man who lives alone in a poorly venti-
the compound benzo[a]pyrene, a chemical carcinogen present lated house without central heating uses a portable unvented
in cigarette smoke, are studied in various tissues. Investigators kerosene heater to warm the house during the winter months.
determine that formation of a secondary metabolite, which One morning, a neighbor finds him in an obtunded state. On
binds covalently to DNA, increases the frequency of lung physical examination, he appears cyanotic. Results of blood
cancers. Genetic polymorphisms are found among persons gas measurement on room air are Po2, 90 mm Hg; Pco2, 35 mm
that affect this frequency. Which of the following is the most Hg; and pH, 7.3. Pulse oximetry shows low oxygen saturation.
likely metabolic pathway for generation of this xenobiotic? Exposure to which of the following is most likely to have
A Biomethylation produced this man’s illness?
B Cytochrome P-450 A Beryllium
C Flavin-containing monooxygenase B Carbon monoxide
D Glucuronidation C Nitrous oxide compounds
E Glutathione reduction D Oxygen
F Peroxidase-dependent cooxidation E Ozone
F Sulfur dioxide
2 An environmental study shows that a gas that exists in
nature is present at increased levels in polluted city air. In the 5 An 8-year-old girl exhibits lethargy and somnolence
upper atmosphere, this gas blocks harmful ultraviolet radiation. with dizziness and weakness after a 6-hour bus ride returning
At ground level, this gas generates free radicals in the lower from a week at summer camp. On physical examination, her
respiratory tract. What is this gas? temperature is 37° C, pulse is 107/min, respiratory rate is 28/
A Argon min, and blood pressure is 130/85 mm Hg. Breath sounds are
B Carbon dioxide audible in all lung fields with no wheezes or crackles. Arte-
C Nitrogen dioxide rial blood gas analysis shows pH, 7.35; Po2, 95 mm Hg; Pco2,
D Ozone 37 mm Hg; and HCO3– 20 mEq/L. Pulse oximetry shows an
E Sulfur dioxide oxygen saturation of 90%, but the spectrophotometrically
measured oxyhemoglobin saturation is 60%. Her blood lactic
3 A textbook author standing knee-deep in a rising tide acid is 8 mmol/L, and total creatine kinase is 445 U/L. She is
next to his home notes that this phenomenon is becoming given 100% oxygen in a hyperbaric chamber and improves in
more frequent. He surmises that the risk for illnesses from in- 20 minutes. She is most likely to have experienced poisoning
fections in populations living in such coastal areas is increas- with which of the following?
ing. Which of the following is most likely to be affected by illness A Aspirin (acetylsalicylic acid)
in these populations? B Carbon monoxide
A Cardiovascular system C Iron sulfate
B Central nervous system D Lead
C Gastrointestinal tract E Methanol
D Hematopoietic system F Organophosphate insecticide
E Respiratory tract
F Urinary tract
114
CHAPTER 9 Environmental and Nutritional Diseases 115

6 Several children between the ages of 3 and 6 years have 10 Increasing numbers of children younger than 5 years are
been admitted to a local hospital because of encephalopathic observed to exhibit cerebral palsy, deafness, blindness, and
crisis. They have lived in the same community all their lives. mental retardation in a coastal community. A coal-burning
All have previously exhibited retarded psychomotor devel- power plant was built in this community 5 years ago. Affected
opment. On physical examination, the children have diffuse families have a high consumption of seafood. Which of the fol-
abdominal pain and are experiencing nausea and vomiting. lowing compounds ingested during pregnancy by the moth-
Head CT scans show marked cerebral edema. Laboratory stud- ers of these children is most likely to be responsible for the
ies show microcytic anemia. An investigator sent to the hous- illness affecting the children?
ing project where the children live finds a rundown apartment A Bisphenol A
complex with extensive water damage, poor plumbing and B Cadmium
ventilation. Toxic exposure to which of the following substanc- C Methyl mercury
es best accounts for these findings? D Organochlorine
A Ethylene glycol E Vinyl chloride
B Kerosene
C Lead 11 A 72-year-old man has had increasing dyspnea for the
D Methanol past year. Decreased breath sounds are heard on auscultation
E Sodium hypochlorite of the right side of the chest. A chest radiograph shows a large
pleural mass that nearly encases the right lung. Exposure to
7 A previously healthy, 27-year-old agricultural worker which of the following metals is most likely to be associated
develops nausea, vomiting, abdominal cramps, cough with with these findings?
wheezing and dyspnea, salivation, and lacrimation 1 hour af- A Arsenic
ter leaving work in a field. On examination, he is afebrile with B Asbestos
a heart rate of 50/min and blood pressure of 90/55 mm Hg. C Beryllium
Laboratory studies show plasma cholinesterase activity less D Chromium
than 50% of normal. Atropine is given intravenously, and his E Nickel
condition improves. He was most likely exposed to which of
the following substances? 12 A 61-year-old woman living in a city with very poor
A Aflatoxin air quality has developed a worsening cough over the past 7
B Cyclodiene months. She has an 80 pack-year history of smoking cigarettes.
C Dioxin One week ago she had an episode of hemoptysis. A chest ra-
D Ergot alkaloid diograph shows a 7-cm infiltrative, perihilar mass in the right
E Organophosphate lung. Exposure to which of the following airborne agents is
F Pyrethrin most likely to be associated with these findings?
A Carbon monoxide
8 An epidemiologic study of a community shows an in- B Nicotine
creased incidence for basal cell and squamous cell carcinomas C Nitrous oxide compounds
in adults. These lesions are distributed predominantly on D Ozone
palms and soles. Analysis of groundwater pumped from wells E Polycyclic aromatic hydrocarbons
for drinking shows increased levels of a heavy metal used in F Dusts containing silica
pressure-treated lumber (with a greenish hue), insecticides, G Sulfur dioxide
and herbicides. Which of the following metals is most likely
implicated by this study? 13 While attending a party, a 19-year-old university stu-
A Arsenic dent drinks 2 L of mixed alcoholic beverages containing 50%
B Beryllium ethanol by volume over 30 minutes. He usually does not drink
C Cadmium much alcohol. His major use of drugs consists of acetamino-
D Lead phen for headaches. Which of the following complications is
E Mercury most likely to prove lethal within the first 12 hours?
A Acute pancreatitis
9 A 36-year-old man is the owner of a radiator repair B Brainstem depression
shop, where he works cleaning, cutting, polishing, and weld- C Hepatic cirrhosis
ing metals. Over 6 months, he develops worsening malaise D Variceal bleeding
with headache and abdominal pains and has difficulty hold- E Wernicke disease
ing his tools. CBC indicates microcytic anemia, and basophilic
stippling of RBCs is seen on the peripheral blood smear. An
elevated blood level of which of the following would be most
useful in determining the toxic exposure causing his illness?
A Alanine aminotransferase
B Calcium
C Creatine kinase
D Selenium
E Zinc protoporphyrin
116 UNIT I General Pathology

14 Ingested ethanol is observed to be metabolized by alco- 18 A 26-year-old woman with a 6-month history of depres-
hol dehydrogenase to an intermediate compound that, upon sion accompanied by active suicidal ideation ingests 35 g of
further metabolism, depletes reduced nicotinamide adenine di- acetaminophen. She quickly experiences nausea and vomiting.
nucleotide (NAD+) in the cytoplasm of hepatocytes. Fatty acid Within 1 day, she becomes progressively obtunded. On physi-
oxidation is subsequently reduced. Population studies show cal examination, her temperature is 36.9° C, pulse is 75/min,
some persons have deficient enzyme activity to metabolize this respirations are 15/min, and blood pressure is 100/65 mm Hg.
intermediate compound. What is this intermediate compound? She is treated with N-acetylcysteine. Depletion of which of the
A Acetaldehyde following is most likely to accentuate her organ damage?
B α-Tocopherol A Glutathione (GSH)
C Formic acid B Amylase
D Glutathione C Creatine kinase
E Hydrogen peroxide D Ketone bodies
E Potassium
15 A case-control study seeks to identify long-term effects
of hormone replacement therapy (HRT) in postmenopausal 19 A 27-year-old, previously healthy man suddenly col-
women receiving exogenous estrogens coupled with proges- lapses at a party where both legal and illicit drugs are being
tins compared with a control group not receiving this therapy. used. En route to the hospital, he requires resuscitation with
The medical records of the women in the study are reviewed defibrillation to establish a normal cardiac rhythm. On physi-
after 20 years. Which of the following complications is most cal examination, his temperature is 40° C; respirations, 30/
likely to be observed in the women receiving HRT? min; heart rate, 110/min; and blood pressure, 175/90 mm Hg.
A Cervical carcinoma He has dilated pupils, a perforated nasal septum, and a promi-
B Chronic ulcerative colitis nent callus on the right thumb. CT scan of the head shows an
C Hepatic cirrhosis acute right frontal lobe hemorrhage. Which of the following
D Pulmonary emphysema substances is most likely responsible for these findings?
E Thromboembolism A Amphetamine
B Barbiturate
16 A 36-year-old woman has been using low-dose estro- C Cocaine
gen–containing oral contraceptives for the past 20 years. She D Ethanol
has smoked one pack of cigarettes per day for the past 18 E Heroin
years. She is G2, P2, and both pregnancies ended with term F Marijuana
live-born infants of low birth weight, but no anomalies. On G Phencyclidine
physical examination, no abnormal findings are noted. Her
BMI is 24. She is at increased risk for developing which of the 20 A 63-year-old man with a history of chronic arthritis
following conditions? has had pronounced tinnitus and episodes of dizziness for
A Breast carcinoma the past 6 months. He has had a headache and nausea for
B Cholecystitis the past 2 days. Physical examination shows he is afebrile
C Dementia with Lewy bodies but has a respiratory rate of 35/min and heart rate of 100/
D Endometrial carcinoma min. He has scattered petechiae over the skin of the upper
E Myocardial infarction extremities. There is no apparent bone conduction or nerve
F Ovarian carcinoma hearing loss. A stool guaiac test result is positive. Labora-
tory studies show serum lactate is 6 mmol/L. An arterial
17 A 22-year-old star football player suddenly collapses blood gas analysis shows a pH of 7.25; PO2, 95 mm Hg; PCO2,
during practice and has a cardiac arrest and cannot be resus- 35 mm Hg; and HCO3–, 15 mEq/L. Acute and chronic toxic-
citated. The medical examiner investigating this sudden death ity from which of the following drugs best explains these
finds marked coronary atherosclerosis and histologic evidence findings?
of hypertension in the renal blood vessels at autopsy. Use of A Acetaminophen
which of the following substances by this man most likely led B Aspirin
to these findings? C Chlorpromazine
A Amphetamine D Iron
B Barbiturate E Morphine
C Benzodiazepine F Quinidine
D Cocaine G Tetracycline
E Ethanol
F Heroin
G Marijuana
CHAPTER 9 Environmental and Nutritional Diseases 117

21 A 20-year-old man is brought to the hospital emergency


department by a friend who found him unconscious in his
apartment after trying to contact him for 3 days. On arrival,
the patient is in a state of respiratory depression. He experi-
ences convulsions for 2 minutes, followed by cardiac arrest.
Advanced cardiac life support measures are instituted, and
he is stabilized and intubated. On physical examination, there
are needle tracks in the left antecubital fossa, miosis, and a
loud diastolic heart murmur. His temperature is 39.2° C. Use
of which of the following substances by this man most likely
produced these findings?
A Cocaine
B Ethanol
C Flurazepam
D Heroin
E Meperidine
F Phencyclidine
G Lysergic acid

22 A clinical trial involves patients with a diagnosis of can-


cer who have intractable nausea as a result of chemotherapy.
The patients are divided into two groups; the group receiving 25 A 14-year-old boy was taken to the emergency depart-
the drug is found to have reduced self-reported nausea and ment of a local hospital, where he died 2 hours later. The
diminished weight loss compared with the placebo group. The principal finding on examination is shown in the figure. Lab-
patients receiving the drug seem to have no major adverse side oratory studies showed that the hematocrit was 17%. Which
effects. Of the following agents, classified as drugs of abuse in of the following terms best describes this injury?
parts of the world, which is most likely to have the beneficial A Blast injury
effects found in this study? B Electrocution injury
A Barbiturate C Gunshot wound
B Cocaine D Incised wound
C Heroin E Laceration
D Marijuana
E Methylphenidate 26 A 33-year-old man incurs thermal burn injuries to 40%
F Methamphetamine of his total body surface area in an accidental fire while repair-
G Phencyclidine ing a fuel storage tank. On physical examination, the skin of
the trunk, neck, and face is pink, shows blister formation, and
23 A 7-year-old boy falls off his bicycle while riding down is painful when touched. The skin of the arms is white and
the street at 5 km/hr. The skin of his right calf and right arm anesthetic. Skin grafting is necessary on the arms, but not on
scrape along the pavement, and the top layer of epidermis other injured areas of skin. Which of the following tissue com-
is removed. Which of the following terms best describes this ponents, when absent, will necessitate skin grafting?
injury? A Collagen fibers
A Abrasion B Dermal appendages
B Burn C Epidermal stratum spinosum
C Contusion D Keratin
D Incision E Macrophages
E Laceration F Nerve endings

24 During a qualifying match for the World Cup, the goal-


keeper is hit in the chest by a soccer ball (football) kicked from
10 m away. He stays in the game. Which of the following in-
jury patterns is most likely to be seen over the chest of the
goalkeeper?
A Abrasion
B Contusion
C Incision
D Laceration
E Puncture
118 UNIT I General Pathology

30 A 30-year-old woman is found dead in her hotel room 3


hours after firemen extinguish a fire on the floor below. Inves-
tigation of the scene shows no signs of fire within her room,
and the medical examiner observes no external findings on
the body. Which of the following conditions best explains the
woman’s death?
A Acute myocardial infarction
B Cerebral hemorrhage
C Malignant hyperthermia
D Pseudomonas aeruginosa septicemia
E Pulmonary edema

31 “It was terribly cold and nearly dark on the last evening
of the old year, and the snow was falling fast. In the cold and
the darkness, a poor little girl, with bare head and naked feet,
roamed through the streets. So the little girl went on with her
little naked feet, which were quite red and blue with the cold.
Shivering with cold and hunger, she crept along.” Which of
27 A 5-year-old boy is brought to the physician for exami- the following pathologic findings in soft tissues is most likely
nation. A police agency suspects child abuse. On physical ex- to be present in this girl?
amination, the child’s skin has the appearance shown in the A Acute inflammation
figure. Which of the following terms best describes this injury? B Apoptosis
A Abrasion C Edema
B Contusion D Hemorrhage
C Hypothermia E Nuclear karyorrhexis
D Kwashiorkor
E Laceration 32 A 76-year-old man has noticed a gradually enlarging
F Thermal burn nodule on the right lower eyelid for 3 years. On physical
examination, the 0.8-cm umbilicated nodule is firm and has
28 While touring the grounds of the Imperial Palace in a small central area of ulceration. The nodule is excised, and
Kyoto, a 75-year-old woman collapses suddenly. She remains plastic repair to the eyelid is performed. Which of the follow-
conscious, but says that she feels weak and light headed. On ing forms of radiation most likely played the greatest role in
physical examination by the nurse, her temperature is 35.2° the development of this man’s lesion?
C, pulse is 93/min, respirations are 17/min, and blood pres- A Gamma rays
sure is 95/50 mm Hg. The temperature in the shade is 34° C B Infrared rays
with 90% humidity. One hour after drinking cool green tea, C Ultraviolet rays
the woman revives and is able to return to her hotel. Which of D Visible rays
the following terms best describes these findings? E X-rays
A Heat cramps
B Heat exhaustion 33 A 56-year-old man with a 60 pack-year history of smok-
C Heat stroke ing has been diagnosed with a squamous cell carcinoma of the
D Malignant hyperthermia larynx. He receives radiation therapy, 40 Sv (4000 cGy) in di-
E Thermal inhalation injury vided doses, to treat the carcinoma. One year later, endoscopy
shows no gross evidence of residual carcinoma. Which of the
29 A 20-year-old man is trying to repair an old electrical appli- following adverse effects is most likely to be present in this
ance in his garage. While testing the function of the appliance, his patient as a result of this radiotherapy?
right hand comes in contact with a frayed electrical cord carry- A Azoospermia
ing 120-V, 10-amp alternating current. Which of the following is B Cerebral atrophy
most likely to develop as a consequence of this electrical injury? C Colonic ulceration
A Bronchoconstriction D Marrow aplasia
B Cerebral artery thrombosis E Vascular fibrosis
C Heat stroke
D Gastric hemorrhage 34 The firemen who initially responded to fight the fires from
E Ventricular fibrillation the Chernobyl nuclear reactor accident were exposed to high ra-
diation levels. Some of the men received doses exceeding 50 Sv
(5000 cGy). Within hours, many became extremely ill. Damage
to which of the following tissues most likely led to this finding?
A Bone marrow
B Cerebrum
C Heart
D Lungs
E Small intestine
CHAPTER 9 Environmental and Nutritional Diseases 119

35 A 3-year-old boy has had a succession of respiratory in-


fections during the past 6 months. On physical examination, the
child appears chronically ill, listless, and underdeveloped. He
is 50% of ideal body weight and has marked muscle wasting.
Laboratory findings include hemoglobin, 9.4 g/dL; hematocrit,
27.9%; MCV, 75 µm3; platelet count, 182,000/mm3; WBC count,
6730/mm3; serum albumin, 4.1 g/dL; total protein, 6.8 g/dL;
glucose, 52 mg/dL; and creatinine, 0.3 mg/dL. Which of the
following is most likely to explain these findings?
A Bulimia
B Folate deficiency
C Kwashiorkor
D Lead poisoning
E Marasmus

36 A 26-year-old woman has had amenorrhea for the past


8 years. She fractured her right wrist 1 year ago after a minor
fall to the ground. On physical examination, she is 175 cm (5
ft 7 in) tall and weighs 52 kg (BMI 17). She has normal sec-
ondary sex characteristics. There are no abnormal findings.
Radiographic measurement of bone density by dual-energy
x-ray absorptiometry shows a bone mineral density that is 1.5
standard deviations below the young adult reference range.
Laboratory findings include anemia and hypoalbuminemia.
Which of the following is the most likely diagnosis?
A Anorexia nervosa
B Bulimia
C Kwashiorkor
D Rickets
E Scurvy

37 A 5-year-old child has had recurrent upper respiratory in-


fections for the past 2 months. The child is at the 55th percentile
for height and the 38th percentile for weight. Physical examina-
tion shows generalized edema, ascites, muscle wasting, and areas 39 A 3-year-old boy does not appear to be developing nor-
of desquamating skin over the trunk and extremities. Laboratory mally. On physical examination, the child has the appearance
studies are most likely to show which of the following findings? shown in the figure. The results from vision testing are nor-
A Abetalipoproteinemia mal. There are no petechiae or areas of purpura on the skin.
B Hypoalbuminemia The abdomen is not enlarged. Which of the following is the
C Hypocalcemia most likely diagnosis?
D Hyperglycemia A Beriberi
E Megaloblastic anemia B Kwashiorkor
C Rickets
38 An epidemiologic study observes increased numbers of D Pellagra
respiratory tract infections among children living in a com- E Scurvy
munity in which most families are at the poverty level. The
infectious agents include Streptococcus pneumoniae, Haemophi- 40 A 3-year-old child has erosion of a roughened corneal
lus influenzae, and Klebsiella pneumoniae. Most of the children surface caused by xerophthalmia. Keratomalacia results in
have had pneumonitis and rubeola infection. The study docu- corneal scarring with eventual blindness after 4 years. This oc-
ments increased rates of keratomalacia, urinary tract calculi, ular damage is now less common because of a United Nations
and generalized papular dermatosis in these children as they initiative to treat a dietary deficiency of which of the following
reach adulthood. These children are most likely to have a defi- nutrients?
ciency of which of the following vitamins? A Iron
A Vitamin A B Niacin
B Vitamin B1 C Protein
C Vitamin E D Vitamin A
D Vitamin D E Vitamin K
E Vitamin K
120 UNIT I General Pathology

41 In a study of lifestyle influences on health, investigators On examination, he exhibits absent deep tendon reflexes, de-
observe that sending children outside to play instead of letting creased vibration and pain sensation, muscle weakness, and
them sit for hours in front of the television can have long-term abnormalities of eye movement. Laboratory studies show he-
health benefits. Which of the following tissues is most like- moglobin, 9.2 g/dL; hematocrit, 27.6%; MCV, 86 µm3; platelet
ly to be in better condition by middle age from this lifestyle count, 208,000/mm3; WBC count, 6080/mm3; total protein,
change? 6.4 g/dL; albumin, 3.4 g/dL; glucose, 70 mg/dL; and creati-
A Bone fractures nine, 0.3 mg/dL. A deficiency of which of the following vita-
B Ocular cataracts mins is most likely to contribute to these findings?
C Urinary tract calculi A Vitamin A
D Pulmonary emphysema B Vitamin B1
E Skin cancers C Vitamin B3
D Vitamin B12
42 A 48-year-old woman injured her right wrist in a fall E Vitamin C
down a flight of stairs. On physical examination, she has F Vitamin E
marked pain on palpation of the wrist and does not want to G Vitamin K
move the hand. A radiograph of the right hand and arm shows
marked osteopenia and a fracture of the radial head. Which of 46 A 52-year-old woman with a long history of ethanol
the following underlying diseases is most likely to contribute abuse has had increasing congestive heart failure for the past
to the risk of fracture in this patient? year. For the past month, she has experienced increasing con-
A Atrophic gastritis fusion, disorientation, and difficulty ambulating. Physical
B Chronic lymphocytic leukemia examination shows nystagmus, ataxia of gait, and decreased
C Coronary atherosclerosis sensation in the lower extremities. Laboratory studies show
D Primary biliary cirrhosis hemoglobin, 13.1 g/dL; hematocrit, 39.3%; MCV, 90 µm3;
E Pulmonary emphysema platelet count, 269,300/mm3; and WBC count 7120/mm3. A
long-term dietary deficiency of which of the following nutri-
43 A 75-year-old woman lives alone and eats sparingly be- ents is most likely to produce these findings?
cause of her low fixed retirement income. For the past 2 weeks, A Folate
she has noticed pain in her right leg. On physical examination, B Niacin
there is marked tenderness to palpation over the lateral aspect C Pyridoxine
of the right shin, a poorly healed cut on the right hand, and a D Riboflavin
diffuse hyperkeratotic skin rash. A radiograph shows a right E Thiamine
tibial diaphyseal subperiosteal hematoma. Laboratory studies
show a hemoglobin level of 11.3 g/dL. A deficiency of which of 47 A 41-year-old woman had a chronic cough for 7 months.
the following nutrients is most likely to explain these findings? She had a positive tuberculin skin test result. A chest radio-
A Ascorbic acid graph showed multiple cavitary lesions in the upper lobes of
B Folate the right and left lungs. She was given isoniazid, rifampin,
C Niacin pyrazinamide, and ethambutol therapy. Now on physical
D Riboflavin examination 6 months later, a peripheral neuropathy is ob-
E Vitamin A served. Administration of which of the following nutrients
F Vitamin K would most likely have prevented the neuropathy?
A Ascorbic acid
44 A 55-year-old woman has had worsening problems B Calciferol
with memory and the ability to carry out tasks of daily living C Calcium
over the past year. She has had watery diarrhea for the past D Cobalamin (vitamin B12)
3 months. Physical examination shows red, scaling skin in E Niacin
sun-exposed areas. Deep tendon reflexes are normal, and sen- F Pyridoxine
sation is intact. Which of the following diseases is she most G Riboflavin
likely to have?
A Beriberi 48 An infant born at term has Apgar scores of 8 and 9 at 1
B Cheilosis and 5 minutes. The infant appears healthy, but 3 days after birth,
C Hypothyroidism there is bleeding from the umbilical cord stump, and ecchymoses
D Marasmus are observed over the buttocks. Seizures soon develop. Which of
E Pellagra the following nutrients is most likely deficient in this infant?
A Folic acid
45 A 9-month-old infant has failure to thrive following a B Iodine
premature birth with low birth weight. The infant has chronic C Iron
cholestatic hepatobiliary disease. The infant is now at the D Vitamin E
40th percentile for height and the 25th percentile for weight. E Vitamin K
CHAPTER 9 Environmental and Nutritional Diseases 121

49 Sir Robert Falcon Scott reaches the South Pole on Janu- 53 A 55-year-old woman has been steadily gaining weight
ary 17, 1912, barely 1 month after Roald Amundsen achieves for the past 30 years. She underwent a cholecystectomy for
this goal with a more experienced and prepared expeditionary cholelithiasis 5 years ago. She does not smoke. She is now 164
party. Scott’s dejected party must now make the long trip back cm (5 ft 4 in) tall and weighs 126 kg (BMI 47). On physical
to their base, but they are weak and running low on supplies, examination, she has decreased range of motion with pain
and the weather is unusually cold, even for Antarctica. Finally, on movement of the knees. Laboratory studies show a serum
they can go no further because of severe storms. Months later, glucose level of 176 mg/dL. This patient is at greatest risk of
a rescue team finds the bodies of the men. All have a hyperker- developing which of the following neoplasms?
atotic, papular rash; ecchymoses; and severe gingival swelling A Colonic adenocarcinoma
with hemorrhages. Which of the following was most likely a B Endometrial carcinoma
contributing cause of death in these men? C Hepatocellular carcinoma
A Beriberi D Pulmonary adenocarcinoma
B Kwashiorkor E Renal cell carcinoma
C Pellagra
D Pernicious anemia 54 In an epidemiologic study of individuals whose BMI is
E Rickets greater than 35, data on lifestyle and disease patterns are col-
F Scurvy lected. Investigators observe that a subset of obese individuals
has a consistently high caloric intake because they lack a feel-
50 An 18-year-old pregnant woman receives no prenatal ing of satiety when eating. These individuals have diminished
care, eats a diet containing mostly carbohydrates and fats, and responsiveness of a hypothalamic receptor for which of the
does not take prenatal vitamins with iron. She feels increas- following molecules?
ingly tired and weak during the third trimester. The infant is A Adenosine
born at 35 weeks’ gestation and is listless during the first week B Glucagon
of life. Laboratory studies show markedly decreased serum C Glucose
ferritin levels in the infant and the mother. Which of the fol- D Insulin
lowing findings from a nutritional deficiency is most likely to E Leptin
be present in both the infant and the mother?
A Dermatitis 55 It is 1:00 am and a hard-working second-year medical
B Diffuse goiter student is intent on finishing her pathology reading assign-
C Microcytic anemia ment. Soon she begins to note that her concentration is fading
D Peripheral neuropathy because 7 hours have passed since she had dinner, and she
E Skeletal deformities is feeling famished. Having studied the chapter on ischemic
F Soft tissue hemorrhages heart disease, she decides to be prudent and forgoes her favor-
ite chocolate cookies, and instead devours two apples, gulping
51 An epidemiologic study evaluates the rate of dental car- them down with a glass of low-fat milk. Of the following sub-
ies and tooth abscesses among children living in communities stances, which one was most likely to have increased rapidly
within a metropolitan area. Investigators discover that the rate when she became hungry and decreased promptly after she
is high among children living in an upper middle class com- finished her healthy snack?
munity, but low in children living in a community below the A α-MSH
poverty level. The levels of trace elements in the water sup- B Corticotropin-releasing factor (CRF)
plies for those communities are measured. A higher level of C Ghrelin
which of the following minerals in the water is most likely to D Leptin
be associated with a lower rate of dental decay among the chil- E Thyrotropin-releasing hormone (TRH)
dren living in the poor community?
A Copper 56 A clinical study of adults with a body mass index of
B Fluoride at least 30 is undertaken. About 8% of these individuals do
C Iodine not have hyperphagia, but are found to have normal levels of
D Selenium leptin and ghrelin, along with a diminished basal metabolic
E Zinc rate. A mutation in which of the following genes is most likely
present in these individuals?
52 A poorly funded epidemiologic study is conducted, and A OB-R
the results appear in a publication available at the supermar- B MC4R
ket checkout counter. The study analyzes the diet of textbook C OB
authors. Which of the following is determined to be the most D POMC
likely dietary deficiency in this population? E PPARγ
A Calcium
B Chocolate
C Folate
D Iron
E Vitamin C
122 UNIT I General Pathology

57 A case-control study of adult men and women is per- concentration is 181 mg/dL. Which of the following is the best
formed to determine the relationship between obesity and dietary advice to give this patient?
cancer. The data indicate an increased risk for cancers of the A Avoid adding salt to food
esophagus and kidney in subjects with a body mass index B Drink more water
above 25. Which of the following substances is most likely to C Increase dietary fiber
contribute to the development of cancer in these subjects? D Reduce intake of saturated fat
A Adiponectin E Take vitamin A supplements
B Aflatoxin
C Insulin-like growth factor 1 (IGF-1) 59 A 40-year-old man notes a family history of colon car-
D Leptin cinoma. He asks his physician how best to reduce his risk of
E Selenium developing this type of cancer. Which of the following dietary
F Trans fats practices should he be advised to follow each day?
A Consume more beef
58 A 45-year-old man, whose mother, father, brother, and B Drink a glass of red wine
uncle all had a history of heart disease, asks his physician C Eat more vegetables
about ways to reduce his risk of developing coronary artery D Have a bowl of ice cream
disease. The patient is 171 cm (5 ft 6 in) tall, weighs 91 kg, E Reduce intake of chocolate
and has a blood pressure of 125/80 mm Hg. His blood glucose

ANSWERS
1 B The cytochrome P-450–dependent monooxygenase, potential for water contamination and gastrointestinal infec-
or mixed function oxidase, system is found in smooth endo- tions. The saltwater kills the mosquitoes, one advantage of
plasmic reticulum, particularly in hepatocytes, and normally the high tide, so arboviral diseases are less likely. The rise in
functions to detoxify endogenous hormones. It also can serve temperature is not enough to significantly affect the cardio-
to activate xenobiotics to carcinogens. Biomethylation by envi- vascular system, except in sporadic but increasingly frequent
ronmental microorganisms of inorganic mercury dumped heat waves. The other listed options are not affected.
into bodies of water can lead to accumulation of toxic methyl PBD9 405–406 BP9 270
mercury, which can work its way up the food chain to humans.
Flavin-containing monooxygenase found in endoplasmic re-
ticulum can oxidize nicotine. Glucuronidation can convert 4 B Heating devices that burn hydrocarbons, such as petro-
naphthylamine to a carcinogen that causes urinary tract can- leum products, generate carbon monoxide, which can build
cers. Reduced glutathione helps to break down free radicals up to dangerous levels in unventilated or poorly ventilated
produced by oxygenase systems such as cytochrome P-450; houses. Chronic carbon monoxide poisoning produces central
xenobiotic metabolism can deplete glutathione and enhance nervous system damage. Carbon monoxide binds much more
free radical cellular injury. The peroxidase-dependent cooxi- tightly to hemoglobin than does oxygen, resulting in hypoxia.
dation pathway can metabolize 2-naphthylamine to a car- Decreased mental functioning generally begins at carboxyhe-
cinogen that causes urinary tract cancers. moglobin levels greater than 20%, and death is likely at levels
PBD9 407 BP9 271 PBD8 402–403 BP8 281 greater than 60%. The classic “cherry red” lividity is rare and
more typical for severe acute poisoning. Acute exposure to be-
ryllium may cause pneumonitis; chronic exposure results in a
2 D Air pollutants form a toxic soup of chemicals that dimi- sarcoid like pulmonary disease. Nitrous oxide compounds and
nish lung function. They are generally tolerated by healthy ozone are found in smog and may cause respiratory discom-
persons, but reduce the quality of life for persons with fort with diminished respiratory function in very young indi-
existing respiratory conditions. Decreasing levels of ozone viduals, very old individuals, and individuals with underlying
in the upper atmosphere have been linked to fluorocarbon respiratory diseases; but such exposure is not immediately life-
release (from refrigerants) and increase the risk for skin can- threatening. In very high concentrations, oxygen can be toxic
cers. Vehicular exhaust is the major contributor to nitrogen to the lungs and can promote diffuse alveolar damage. Sulfur
dioxide and ozone emissions that react to form smog. Argon dioxide emissions are a component of smog and promote acid
is a nonreactive noble gas that constitutes 1% of the Earth’s rain; they may increase the risk of chronic bronchitis.
atmosphere. Carbon dioxide is a greenhouse gas that is PBD9 409 BP9 273 PBD8 411 BP8 282–283
increasingly contributing to global climate change, but more
is breathed out than is in the atmosphere. Sulfur dioxide
released from industrial processes and burning coal forms 5 B Blame the bus. Carbon monoxide (CO) binds to hemo-
acid rain and can contribute to chronic obstructive pulmo- globin 200 times more avidly than does oxygen, leading to
nary disease. reduced oxygen saturation; CO binds to cardiac myoglobin
PBD9 407–409 BP9 272–273 even more avidly than to hemoglobin. Tissue hypoxia results
in lactic acidosis. Cardiac and skeletal muscle begin to break
down, releasing creatine kinase. The brain requires high lev-
3 C Climate change is bringing challenges for public els of oxygen delivery, so neurologic findings are often the
health. In coastal regions, more flooding leads to increased first signs of CO poisoning. The CO toxicity is exacerbated
CHAPTER 9 Environmental and Nutritional Diseases 123

by exercise. The other listed options do not account for the lead poisoning, in anemia of chronic disease, and in iron
decreased oxygen saturation. Aspirin poisoning can lead to deficiency anemia. Lead interferes with heme biosynthesis
metabolic acidosis. Iron toxicity leads to nausea and abdomi- and inhibits the incorporation of iron into heme; as a result,
nal pain with fluid loss and hypovolemia, metabolic acido- zinc is used instead. Hepatic damage with elevation of liver
sis, and hyperglycemia. Lead poisoning is usually chronic enzymes ALT and AST is not a major feature of lead poison-
and results in encephalopathy, anemia, and abdominal pain. ing, but acute increases in these enzymes could be seen with
Methanol is metabolized to formic acid and formaldehyde, acetaminophen toxicity. The muscle enzyme creatine kinase
causing a metabolic acidosis and damage to the central ner- is not elevated because muscle is not directly damaged by
vous system and eye. Organophosphates are irreversible lead, although a neuropathy can occur. Lead can damage
cholinesterase inhibitors that produce acute neurotoxicity. renal tubules and cause renal failure, but specific alterations
PBD9 409 BP9 273 PBD8 405 BP8 282–283 in electrolytes with decreased calcium are not specific for
lead-induced renal failure. Selenium is a micronutrient.
PBD9 410–412 BP9 274–275 PBD8 406–407 BP8 283–285
6 C Old flaking paint that is lead-based has a sweet taste,
attracting small children to ingest it. The major risk to children
from lead ingestion is neurologic damage. Venous blood lead 10 C Methyl mercury is toxic to the developing brain. It
levels should normally be less than 10 µg/dL. Ethylene glycol is incorporated into organic compounds within bacteria and
is found in antifreeze and can produce acute renal tubular necro- works its way up the food chain to humans. Victims of cadmi-
sis. Kerosene, a hydrocarbon used in some household heating um poisoning primarily exhibit complications involving kid-
devices, can generate carbon monoxide fumes if not properly ney and lung. Bisphenol A is used in the manufacture of epoxy
ventilated and can cause gastrointestinal and respiratory toxic- resins found in nearly every bottle and can; it is an endocrine
ity when ingested. Methanol ingestion can cause acute central disruptor. Organochlorines such as the insecticide DDT also
nervous system depression, acidosis, and blindness. House- get into the food chain and mainly disrupt endocrine metabo-
hold bleach (sodium hypochlorite) is a local irritant and is not lism. Non-pesticide organochlorines include polychlorinated
likely to be found in the living conditions of these children. biphenyls (PCBs) that can cause chloracne, and an initial
PBD9 410–412 BP9 274–275 PBD8 406–407 BP8 283–285 target of the environmental movement spearheaded by Pete
Seeger sailing the Clearwater along the Hudson River. Vinyl
chloride used in the manufacture of polyvinyl chloride plastics
7 E The muscarinic effects of acute organophosphate poi- is linked to hepatic angiosarcomas.
soning are counteracted by the atropine. Organophosphates PBD9 412 BP9 275 PBD8 1329 BP8 285
are powerful pesticides with neurotoxicity to humans. In con-
trast, cyclodienes, such as DDT, have low toxicity to humans,
but build up in the environment, working their way up the 11 B Asbestos fibers can cause pulmonary interstitial fibrosis
food chain. Dioxins are defoliants that are potentially im- with restrictive lung disease, and there is an increased risk of
munosuppressive, carcinogenic, and teratogenic. Pyrethrins malignancy. Individuals who have been exposed to asbestos
(derived from chrysanthemum flowers) are pesticides with and who smoke have a greatly increased incidence of bron-
weak toxicity, often used in aircraft cabins upon landing to chogenic carcinoma. Mesothelioma is uncommon, even in in-
knock out any stowaway insects, but not human passengers. dividuals with asbestos exposure, but virtually all occurrences
Aflatoxins and ergot alkaloids are naturally occurring toxins; of malignant mesothelioma are in individuals who have been
the former are found in moldy cereal and are a risk for hepa- exposed to asbestos. Arsenic exposure is a risk factor for skin
tocellular carcinoma, whereas the latter are found in moldy cancer. Chronic beryllium exposure may lead to sarcoid like
grains and can induce seizure activity. granuloma formation. Chromium exposure increases the risk
PBD9 413–414 BP9 276–277 PBD8 400 BP8 287 of carcinomas of the upper respiratory tract and lung. Nickel
exposure is associated with cancers of the respiratory tract.
PBD9 414, 690–692 BP9 277, 477–478 PBD8 409–410 BP8 286–288
8 A Chronic exposure to arsenic-containing compounds
can increase the risk for nonmelanoma skin cancers in a
distribution different from those linked to the most common 12 E The infiltrative perihilar mass suggests lung cancer.
cause, ultraviolet light exposure. Beryllium inhaled as a gas Polycyclic hydrocarbons and nitrosamines, found in tobacco
can produce an acute toxicity to the respiratory tract; chronic smoke, are the key contributors to the development of lung
exposure can produce sarcoid like granulomas. Cadmium cancer. The carbon monoxide levels of smokers are increased,
toxicity can lead to renal tubular necrosis, osteopenia, and but this promotes hypoxemia, not cancer. The nicotine in ciga-
chronic pulmonary disease. Lead toxicity acutely produces rette smoke has a stimulant effect on the central nervous sys-
renal and gastrointestinal damage, and chronically it causes tem, but it does not play a major role in cancer development.
anemia, neuropathy, and encephalopathy. Mercury poisoning Nitrous oxide compounds and ozone are found in smog, but
leads to central nervous system damage. they are not closely associated with the risk of lung cancer. Sili-
PBD9 412 BP9 275–276 PBD8 408 BP8 285 ca dust exposure slightly increases the risk of lung cancer, most
often in individuals with prolonged occupational exposure. Sul-
fur dioxide emissions are a component of smog and promote
9 E This man has experienced occupational exposure to acid rain; they may increase the risk of chronic bronchitis.
lead and shows symptoms and signs of lead toxicity. The PBD9 414–417 BP9 277–279 PBD8 411 BP8 289–290
concentration of zinc protoporphyrin is elevated in chronic
124 UNIT I General Pathology

13 B A large amount of ethanol ingested over a short time 17 D Cocaine is a powerful vasoconstrictor, and the cardiac
can elevate blood ethanol to toxic levels with CNS depres- complications of its use include acute arterial vasoconstric-
sion because the alcohol dehydrogenase in liver metabolizes tion with ischemic injury and arrhythmias. Atherosclerosis,
ethanol by zero-order kinetics. The combination of acetamin- affecting small, peripheral branches of the coronary arteries,
ophen and ethanol increases the likelihood of hepatic toxicity can be marked with chronic use. Amphetamines may also in-
with hepatic necrosis and acute liver failure. Pancreatitis is a duce cardiac arrhythmias. Barbiturates are depressants and
potential complication of chronic ethanol abuse. Cirrhosis is can cause respiratory failure. Benzodiazepines can produce
a long-term complication of chronic ethanolism. Hemateme- respiratory failure. Acute ethanol poisoning can cause cen-
sis from gastritis and gastric ulceration is more typically seen tral nervous system depression and coma. Heroin overdoses
with chronic ethanolism, and variceal bleeding is a compli- can be accompanied by pulmonary edema and respiratory
cation of hepatic cirrhosis. Wernicke disease occurs rarely, failure. Marijuana is a minor tranquilizer; no serious physi-
even in alcoholics, and probably results from concomitant ologic effects are associated with its use.
chronic thiamine deficiency. PBD9 423–424 BP9 284–285 PBD8 417–418 BP8 295–296
PBD9 417–419, 422 BP9 280–281 PBD8 412–414 BP8 290–292

18 A Acetaminophen toxicity leads to hepatic necrosis, in-


14 A Acetaldehyde contributes to acute toxicity of ethanol. dicated by rising ALT and AST levels. If death is not imme-
In some populations, such as Asian, there is a variant form of diate, hyperbilirubinemia also can be seen. N-Acetylcysteine
acetaldehyde dehydrogenase that is less effective at metabo- augments glutathione by contributing a sulfhydryl group for
lizing acetaldehyde to acetic acid. α-Tocopherol is vitamin E, binding to toxic metabolites. Elevated serum amylase is seen
an antioxidant that protects cells from injury. Formic acid is in pancreatitis. Elevated serum creatine kinase is seen with
one of the toxic metabolites, along with formaldehyde, that injury to skeletal and cardiac muscle. Ketonuria is a feature
are produced when methanol is metabolized by alcohol dehy- of absolute insulin deficiency in diabetes mellitus; it also is a
drogenase. Glutathione is an antioxidant that protects cells feature of starvation. Hypokalemia can be a feature of renal
from free radical injury. Hydrogen peroxide is metabolized to diseases and of glucocorticoid deficiency.
water in peroxisomes as ethanol is converted to acetalde- PBD9 422 BP9 284 PBD8 416–417 BP8 294, 297
hyde by alcohol dehydrogenase.
PBD9 418–419 BP9 280–281 PBD8 413–414 BP8 290–292
19 C Cocaine is a powerful vasoconstrictor and has vari-
ous vascular effects, including ischemic injury to the nasal
15 E Hormone replacement therapy (HRT) increases septum following the route of administration, which is inha-
the risk of thromboembolic disease, as is the case with oral lation in this case. Many complications result from the car-
contraceptive therapy. HRT was long considered to have a diovascular effects, which include arterial vasoconstriction
protective effect against cardiovascular disease because ex- with ischemic injury to the heart, arrhythmias, and central
ogenous estrogens increase HDL and decrease LDL. This is nervous system (CNS) hemorrhages. Hyperthermia is another
not true of all women, however, and progestins tend to have complication with “excited delirium” in some cases of cocaine
the opposite effect. The risk of cervical carcinoma is more intoxication. The callus is caused by flicking a lighter for use
closely related to a lifestyle that increases the likelihood of of a crack cocaine pipe. Amphetamines are CNS stimulants.
human papillomavirus infection. Inflammatory bowel dis- Barbiturates are CNS depressants. Acute ethanolism may
ease is not associated with HRT. Hepatic cirrhosis in men lead to CNS depression, but it does not have serious immedi-
may cause decreased degradation of circulating estrogens, ate cardiac effects. Opiates can depress CNS and respiratory
leading to testicular atrophy. Pulmonary emphysema typi- function, and heroin may produce acute pulmonary edema.
cally is related to cigarette smoking, not to HRT. Marijuana (with active agent delta-9-tetrahydrocannabinol)
PBD9 421 BP9 282–283 PBD8 414–415 BP8 292 has no serious acute toxicities. Phencyclidine (PCP) produces
an acute toxicity that mimics psychosis.
PBD9 423–424 BP9 284–285 PBD8 417–418 BP8 295–296
16 E With low estrogen–containing oral contraceptives
currently in use, there is no increase in risk for coronary ath-
erosclerosis or myocardial infarction in nonsmoking women 20 B Chronic aspirin (acetylsalicylic acid) toxicity (<3 g/
younger than 45 years. The risk is increased, however, in day) can result in various neurologic problems. Aspirin also
women older than 35 years who smoke. Smoking during inhibits platelet function by suppressing the production of
pregnancy increases the likelihood for low-birth-weight in- thromboxane A2, promoting bleeding. Acute aspirin toxicity
fants. Oral contraceptives do not increase the risk for breast can cause metabolic acidosis with respiratory compensation.
cancer, and they decrease the likelihood of ovarian and en- Acute toxicity initially produces a pure respiratory alkalosis
dometrial cancer; the risk for these latter two cancers is in- due to stimulation of medullary respiratory centers. Increas-
creased by postmenopausal hormone replacement therapy ing absorption of the salicylic acid drives metabolic acidosis,
(HRT). The risk for cholecystitis increases with postmeno- further exacerbated by poisoning of mitochondrial oxidative
pausal HRT. There is no evidence for risk of dementia with phosphorylation leading to anaerobic glycolysis and lactic
oral contraceptives or HRT. acidosis. Acetaminophen toxicity primarily targets the liver.
PBD9 421 BP9 283 PBD8 415 BP8 292 Chlorpromazine causes cholestatic jaundice. Iron toxicity
typically has a component of gastroenteritis with fluid and
CHAPTER 9 Environmental and Nutritional Diseases 125

blood loss with hypovolemia. Morphine and other opiates unburned gunpowder. A blast injury can produce various
tend to produce respiratory depression with respiratory findings, depending on what struck the body or what the
acidosis. Quinidine therapy may lead to hemolytic anemia. body struck, but the focal stippling seen in the figure is not
Tetracycline can discolor the teeth of children; it can lead to likely to be present. An electrocution injury often produces
phototoxicity with sunburn. minimal findings; the site of entry of the current may be quite
PBD9 422 BP9 284 PBD8 417 BP8 294, 297 small. An incised wound is made by a sharp instrument that
produces clean-cut edges, whereas a laceration is an irregular
disruption in the skin and has torn edges.
21 D Heroin is an opiate narcotic that is a derivative of mor- AP3 eFig. 16-24 PBD9 426 BP9 287–288 PBD8 420 BP8 297–298
phine. Opiates are central nervous system (CNS) depressants,
and overdoses are accompanied by respiratory depression,
convulsions, and cardiac arrest. The typical mode of admin- 26 B The patient has a full-thickness burn injury to the
istration is by injection. An infection, such as an endocarditis arms, but only partial-thickness burns of other areas. In a
explaining his heart murmur, often results from such use be- full-thickness burn, all structures from which reepithelializa-
cause nonsterile injection technique is employed. Cocaine is tion could occur are lost, including dermal appendages such
most often inhaled rather than injected, and acutely produces as sweat glands and hair follicles. Loss of only the epidermal
a state of excited delirium. Ethanol is typically ingested and, basal layer would not prevent reepithelialization from the
in excess, can lead to coma and death. Flurazepam is most of- skin appendages. Fibroblasts can produce more collagen,
ten ingested; excessive use can lead to respiratory depression. although elastic fibers do not regenerate; this is why burned
Meperidine is an analgesic that can cause respiratory depres- skin tends to lose its elasticity, requiring additional grafting
sion and bradycardia in overdosage. Phencyclidine (PCP) is a procedures in growing children. The superficial layer of
schizophrenomimetic and is usually ingested; users have a his- keratin serves a protective function. Its loss in burns increases
tory of erratic behavior. Lysergic acid (LSD) is a hallucinogen. fluid and electrolyte loss, but it reforms if reepithelialization
PBD9 424 BP9 285–286 PBD8 418–419 BP8 296 occurs. Blood monocytes can migrate into tissues to become
macrophages, which help in remodeling the damaged tissues.
Loss of the nerve endings in full-thickness burns leads to the
22 D The medical uses of marijuana include intractable loss of sensation noted on physical examination in this patient,
nausea and glaucoma. Marijuana, with the active substance but this process does not govern reepithelialization.
tetrahydrocannabinol (THC), has a sedative effect on the AP3 Figs. 16-108, 16-109 PBD9 426–427 BP9 288
central nervous system. Barbiturates are sedatives, but have PBD8 421–422 BP8 298–299
no effect on severe nausea. Cocaine, methylphenidate, and
methamphetamine are stimulants. Heroin is an opioid; mor-
phine sulfate, an opioid derivative, has a beneficial effect in 27 A The figure shows superficial tears in the epidermis,
treating intractable pain in cancer patients. Phencyclidine with underlying superficial dermal hemorrhage, typical of
(PCP) has a schizophrenomimetic effect. abrasions made by a scraping type of injury. A contusion is a
PBD9 425 BP9 286–287 PBD8 419 BP8 296–297 bruise characterized by breakage of small dermal blood ves-
sels and bleeding. Hypothermia (exposure to cold) has no
characteristic appearance. Kwashiorkor, resulting from lack
23 A A scraping injury produces an abrasion, but does of protein in the diet, may lead to flaking of the skin and
not break through the skin. A burn injury causes coagula- irregular pigmentation. A laceration is a cut with torn edges.
tive necrosis without mechanical disruption. A contusion is Thermal burns may produce erythema, blistering, and cracking
a bruise with extravasation of blood into soft tissues. Lacera- of skin, but not a scraped appearance.
tions break the skin or other organs in an irregular pattern. AP3 Fig. 16-102 PBD9 426 BP9 287–288 PBD8 420–421
An incised wound is made with a sharp instrument such as a BP8 297–298
knife, leaving clean edges.
AP3 Fig. 16-102 PBD9 426 BP9 287–288 PBD8 420–421
BP8 297–298 28 B Heat exhaustion results from failure of the cardio-
vascular system to compensate for hypovolemia caused
by body water depletion. It is readily reversible by replac-
24 B A blow with a blunt object produces soft tissue ing lost intravascular volume. Vigorous exercise with
hemorrhage without breaking the skin. An abrasion scrapes electrolyte loss can produce muscle cramping typical of
away the superficial epidermis. A laceration is an irregular heat cramps. Heat stroke is associated with organ damage
tear in the skin or other organ. An incised wound is made when core body temperature rises above 36° C. Malignant
by a sharp object and is longer than a puncture, which has a hyperthermia occurs from a metabolic derangement such
rounded outline and is deeper than it is wide. as thyroid storm, or from administration of certain drugs
AP3 Fig. 16-101 PBD9 426 BP9 287–288 PBD8 420 BP8 297–298 such as succinylcholine; it may also result from excited
delirium associated with cocaine use. Inhalation injury is
seen when a fire occurs in an enclosed space, and hot, toxic
25 C The figure shows the entry site of a gunshot wound gases are inhaled.
made at close range. There is a sharply demarcated skin PBD9 427 BP9 289 PBD8 422 BP8 299
defect, and the surrounding skin shows some stippling of
126 UNIT I General Pathology

29 E Electrical current, especially alternating current, dis- matory reaction. With time, these vessels undergo fibrosis
rupts nerve conduction and electrical impulses, particularly and severe luminal narrowing. There is ischemia of the sur-
in the heart and brain. Heart function is often affected when rounding tissue and formation of a scar. The radiation used
electrical current passes through the body to ground. This in therapeutic dosages is carefully delivered in a limited field
can lead to severe arrhythmias, especially ventricular fibril- to promote maximal tumor damage, while reducing damage
lation. These are immediate effects. The amount of tissue in- to surrounding tissues. Whole-body irradiation affects the
jury from standard (U.S.) household current is generally not bone marrow, gonads, gastrointestinal tract, and brain, but
great, and there may be just a small thermal injury at the site therapeutic radiation is carefully focused on the neoplasm to
of entry or exit of the current on the skin. The other listed op- prevent widespread tissue damage.
tions are not a direct consequence of an electrical injury. PBD9 428–432 BP9 290–292 PBD8 423–425 BP8 300–303
PBD9 427–428 BP9 289 PBD8 422 BP8 299–300

34 B These firemen are true heroes whose selfless actions


30 E Fires in enclosed spaces produce hot, toxic gases. prevented a much greater disaster. The cerebral syndrome
The inhalation of these gases can lead to death from pulmo- occurs within hours in individuals exposed to a massive total-
nary edema even when there is no injury from flames. An body radiation dose. Doses of >2 Sv (>200 cGy) can be fatal
acute myocardial infarction is possible, but not probable in because of injury to radiosensitive marrow and the gastroin-
a woman of this age. Cerebral hemorrhage is more likely testinal tract, but death occurs after days to weeks, not hours.
due to trauma, such as a fall or blow to the head. Malignant Cardiac and skeletal muscle tissue is relatively radioresis-
hyperthermia, which occurs when core body temperature is tant. Early findings in radiation-induced lung injury include
greater than 40° C, is produced by metabolic disorders such edema; interstitial fibrosis develops over years in individuals
as hyperthyroidism and drugs such as succinylcholine and who survive the injury.
cocaine. Infections occur days to weeks after a burn injury PBD9 430–431 BP9 292–293 PBD8 423–426 BP8 303–304
because of the loss of an epithelial barrier to infectious agents.
PBD9 427 BP9 288–289 PBD8 421 BP8 298
35 E Body weight less than 60% of normal with muscle
wasting is consistent with marasmus, a form of protein en-
31 C The short story “The Little Match Girl” by Hans ergy malnutrition that results from a marked decrease in to-
Christian Andersen provides a description of hypothermia. tal caloric intake. In kwashiorkor, protein intake is reduced
Slowly developing and prolonged exposure to cold leads to more than total caloric intake, and body weight is usually
peripheral vasoconstriction along with edema caused by in- 60% to 80% of normal. Hypoalbuminemia is a key labora-
creased vascular permeability. As her core body temperature tory finding. Malignancies can promote wasting, but not to
dropped to 32.2° C (90° F), she had involuntary shivering and this degree. This child’s problems are far more serious than
then envisioned her loving grandmother just before loss of a single vitamin deficiency; a lack of folate could account for
consciousness, followed by bradycardia. Death ensues before the child’s anemia, but not for the wasting. Bulimia is an eat-
an inflammatory response to cellular injury can occur. Apop- ing disorder of adolescents and adults that is characterized
tosis is single cell necrosis, and the hypothermia described in by binge eating and self-induced vomiting. Lead poisoning
the story would affect large regions of the body. Though there can lead to anemia and encephalopathy, but it does not cause
is vascular permeability, vascular integrity prevents hemor- severe wasting.
rhage. Soft tissues with low metabolic rate are resistant to PBD9 433–434 BP9 294 PBD8 428–429 BP8 304–306
ischemia and infarction, so findings of coagulative necrosis,
such as nuclear fragmentation, are unlikely to be seen acutely.
PBD9 427 BP9 289 PBD8 422 BP8 299 36 A The decreased food intake from self-imposed di-
eting in a woman can lead to changes such as hormonal
deficiencies (e.g., follicle-stimulating hormone, lutein-
32 C Exposure to sunlight is a risk factor for developing ma- izing hormone, thyroxine). The result is diminished es-
lignancies involving the skin (basal cell carcinoma, squamous trogen synthesis, which promotes osteoporosis, as in the
cell carcinoma, and malignant melanoma). Ultraviolet (UV) postmenopausal state. Bulimia with binging and purg-
rays, mainly the UVB component, are the major causative agent ing can be accompanied by electrolyte disturbances, and
of these malignancies. Infrared radiation causes mainly thermal weight tends to be maintained in most cases. Kwashiorkor
injury. Visible light has minimal effects. The ambient x-radia- is a disease mainly of children who have reduced protein
tion and gamma rays that filter through the earth’s atmosphere intake. Rickets is a specific deficiency of vitamin D that
are minimal and have no significant health effects. X-radiation causes skeletal deformities in children. Scurvy, which re-
from radiologic imaging is certainly a concern when patients sults from vitamin C deficiency, does not affect hormonal
receive multiple imaging studies, but the cumulative dosages function.
remain small. PBD9 435 BP9 295–296 PBD8 427 BP8 304, 306
PBD9 1155 BP9 863–864 PBD8 404 BP8 309–312

37 B The findings are consistent with kwashiorkor, a nu-


33 E Therapeutic doses of radiation can cause acute vascu- tritional disorder predominantly of decreased protein in the
lar injury, manifested by endothelial damage and an inflam- diet. Hypoalbuminemia is characteristic of this condition.
CHAPTER 9 Environmental and Nutritional Diseases 127

Abetalipoproteinemia is a rare disorder that causes vitamin and calcium help build and maintain growing bone. Exercise
E deficiency. Hypocalcemia can occur as a consequence of helps build bone mass, which protects against osteoporo-
vitamin D deficiency. Hyperglycemia occurs in diabetes sis later in life, particularly in women. Renal function is not
mellitus; the wasting associated with this disease affects adi- greatly affected by environment. There are some deleterious
pose tissue and muscle, and edema is not a feature. Megalo- effects on the eye (cataracts) and the skin (cancer, elastosis)
blastic anemia is a feature of specific deficiencies of vitamin from increased exposure to UV radiation in sunlight. In-
B12 or folate. creased air pollution in many cities has led to an increased
PBD9 434–435 BP9 294–295 PBD8 428–429 BP8 304–306 incidence of pulmonary diseases, and children are particu-
larly at risk.
PBD9 439–441 BP9 298–300 PBD8 433–435 BP8 309–312
38 A Vitamin A is important in maintaining epithelial sur-
faces. Deficiency of this vitamin can lead to squamous meta-
plasia of respiratory epithelium, predisposing to infection. 42 D The osteopenia in this patient can result from osteo-
Increased keratin buildup leads to follicular plugging and pap- malacia, the adult form of vitamin D deficiency. Vitamin D
ular dermatosis. Desquamated keratinaceous debris in the uri- is a fat-soluble vitamin, and it requires fat absorption, which
nary tract forms the nidus for stones. Ocular complications of can be impaired by chronic cholestatic liver disease, biliary
vitamin A deficiency include xerophthalmia and corneal scar- tract disease, and pancreatic disease. Atrophic gastritis affects
ring, which can lead to blindness. Vitamin B1 (thiamine) defi- vitamin B12 absorption. Leukemias do not tend to erode
ciency causes problems such as Wernicke disease, neuropathy, bone. Heart disease caused by atherosclerosis does not affect
and cardiomyopathy. Vitamin D deficiency in children causes bone density. Emphysema can result in a hypertrophic osteo-
rickets, characterized by bone deformities. Vitamin E deficien- arthropathy, but not osteopenia.
cy occurs rarely; it causes neurologic symptoms related to de- PBD9 439–441 BP9 298–300 PBD8 435–436 BP8 310–312
generation of the axons in the posterior columns of the spinal
cord. Vitamin K deficiency can result in a bleeding diathesis.
PBD9 436–438 BP9 296–298 PBD8 431–433 BP8 306–309 43 A Signs and symptoms of scurvy can be subtle. The
diet must contain a constant supply of vitamin C (ascorbic
acid) because none is produced endogenously. Older indi-
39 C Rickets, which is caused by vitamin D deficiency, is viduals with an inadequate diet are as much at risk as young-
characterized by skeletal deformity such as the bowing of er individuals. Folate deficiency can lead to anemia, but it
the legs seen in this boy. Lack of bone mineralization (os- does not cause capillary fragility with hematoma formation
teopenia) leads to this deformation. Beriberi, from thiamine or skin rash. Niacin deficiency can lead to an erythematous
deficiency, can result in heart failure and peripheral edema. skin rash in sun-exposed areas, but not to anemia. Ribofla-
A diet containing insufficient protein can result in kwashi- vin deficiency can lead to findings such as glossitis, cheilosis,
orkor, characterized by areas of flaking, depigmented skin. and neuropathy. Vitamin A deficiency can produce a skin
Pellagra, resulting from niacin deficiency, is characterized rash, but it does not cause anemia. Vitamin K is important in
by dermatitis in sun-exposed areas of skin. Scurvy, resulting maintaining proper coagulation, but a deficiency state is not
from vitamin C deficiency, can produce bone deformities, associated with anemia or skin rash.
particularly at the epiphyses, because of abnormal bone ma- PBD9 442–443 BP9 301–302 PBD8 437 BP8 312
trix, not abnormal calcification. The absence of hemorrhages
in this child makes this unlikely, however.
PBD9 438–442 BP9 298–299 PBD8 433–436 BP8 309–312 44 E Pellagra is caused by a deficiency of niacin. The clas-
sic presentation includes the “3 D's”: diarrhea, dermatitis,
and dementia. Beriberi, resulting from thiamine (vitamin
40 D Vitamin A is essential to maintain epithelia. The lack B1) deficiency, can lead to heart failure, neuropathy, and Wer-
of vitamin A affects the function of lacrimal glands and con- nicke disease. Cheilosis describes the fissuring at the corners
junctival epithelium, promoting keratomalacia. Dr. Alfred of the mouth that accompanies riboflavin (vitamin B2) de-
Sommer’s research convinced UNICEF that it would cost ficiency. In hypothyroidism, which could be due to iodine
just pennies per child to eliminate vitamin A deficiency in deficiency, the skin tends to be coarse and dry. Marasmus
250 million children on earth. Iron is essential for produc- describes the severe wasting that occurs in individuals with
tion of heme, which is needed to manufacture hemoglobin a diet that is markedly deficient in all nutrients.
in RBCs. Niacin is involved with nicotinamide in many PBD9 442 BP9 302 PBD8 438 BP8 306, 314
metabolic pathways, and deficiency leads to diarrhea, der-
matitis, and dementia. Dietary protein is essential for build-
ing tissues, particularly muscle, but it has no specific effect 45 F Vitamin E deficiency is uncommon, but it may be
in maintaining ocular structures. Vitamin K is beneficial seen in low-birth-weight infants with poor hepatic func-
for synthesis of coagulation factors by the liver to prevent tion and fat malabsorption. The neurologic manifestations
bleeding problems. are similar to those seen in vitamin B12 deficiency; affected
PBD9 437–438 BP9 296–298 PBD8 431–433 BP8 306–309 infants may have anemia, but it is not of the megaloblastic
type. Vitamin A deficiency in infants and children can lead to
blindness from keratomalacia. It is the most common cause
41 A Vitamin D can be synthesized endogenously in skin of preventable blindness in this population group. Vitamin B1
with exposure to ultraviolet (UV) light. Together, vitamin D (thiamine) deficiency can lead to beriberi. Vitamin B3 (niacin)
128 UNIT I General Pathology

deficiency can lead to pellagra. Vitamin C deficiency leads to 50 C Iron deficiency, which gives rise to microcytic ane-
scurvy, which can be accompanied by anemia from bleeding mia, is common in women of reproductive age because of
and from decreased iron absorption. Vitamin K deficiency menstrual blood loss and in children with a poor diet. Dur-
leads to bleeding problems. ing pregnancy, women have greatly increased iron needs.
PBD9 442–443 BP9 302 PBD8 438 BP8 314 Low serum ferritin is indicative of iron deficiency. Dermati-
tis can be seen in pellagra (niacin deficiency). Goiter results
from iodine deficiency, but this is a rare occurrence today
46 E Individuals with a history of chronic alcoholism are of- because of newborn testing and widely available foods with
ten deficient in thiamine and other nutrients (ethanol provides iodine. Peripheral neuropathy is more characteristic of beri-
empty calories). Thiamine deficiency can lead to neuropathy, beri (thiamine deficiency) and deficiencies in riboflavin (vitamin
cardiomyopathy, and Wernicke disease. Alcoholic individuals B2) and pyridoxine (vitamin B6). Bowing of the long bones
often have folate deficiency, with resultant macrocytic anemia, and epiphyseal widening can be seen in rickets (vitamin D
but this finding is not present here. Niacin deficiency leads to deficiency). Soft tissue hemorrhages can be seen in scurvy
pellagra. Pyridoxine and riboflavin deficiencies can lead to (vitamin C deficiency).
neuropathy, but do not produce cerebral findings. PBD9 443 BP9 303 PBD8 439, 659 BP8 304, 435–436
PBD9 419, 433, 442 BP9 302 PBD8 414, 438 BP8 290–292

51 B Water in some areas naturally contains fluoride,


47 F Isoniazid is a pyridoxine (vitamin B6) antagonist. In- and dental problems in children are fewer in these areas be-
dividuals receiving isoniazid therapy for tuberculosis may cause tooth enamel is strengthened. Fluoride can be added
need supplementation to prevent vitamin B6 deficiency. to drinking water, but opposition to this practice, from ig-
Ascorbic acid (vitamin C) is antiscorbutic (prevents scurvy). norance or fear, is common. Copper deficiency can produce
Calciferol (vitamin D) helps maintain calcium levels. Calcium neurologic defects. Iodine deficiency can predispose to thy-
intake helps maintain bone mass and serum calcium level; roid goiter. Selenium is a trace mineral that forms a compo-
hypocalcemia can lead to neural excitability with muscular nent of glutathione peroxidase; deficiency may be associated
contractions. Cobalamin (vitamin B12) deficiency may pro- with myopathy and heart disease. Serious illnesses from
duce a macrocytic anemia and a peripheral neuropathy, but trace element deficiencies are rare. Zinc is a trace mineral that
it does not result from isoniazid therapy. Niacin deficiency aids in wound healing; a deficiency state can lead to stunted
causes pellagra (diarrhea, dermatitis, dementia). Riboflavin growth in children and a vesicular, erythematous rash.
deficiency may produce neuropathy, glossitis, and cheilosis. PBD9 443 BP9 303 PBD8 439 BP8 315
PBD9 433, 442 BP9 302 PBD8 438 BP8 314

52 B This vignette is just as imaginative as those appear-


48 E Coagulation factors II, VII, IX, and X synthesized ing in many public media sources, so beware the claims and
by the liver require vitamin K for their production. Hemor- apply principles of evidence-based medicine. Some would
rhagic disease of the newborn can occur in infants who lack agree that there is never quite enough chocolate, and much
sufficient intestinal bacterial flora to produce this nutrient. of the world’s population must get by without it. However,
Breastfeeding transiently potentiates this effect, because Lac- countries with the highest chocolate consumption have pro-
tobacillus found in breast milk does not synthesize vitamin duced the most Nobel laureates! Serious dietitians would
K. Routine intramuscular injection of vitamin K soon after probably choose option A (iron deficiency), which is a de-
birth prevents this complication. Iron deficiency leads to ane- ficiency most likely to be seen in menstruating women, in
mia, not to bleeding. Vitamin E is an antioxidant and is rarely pregnant women, and in children. Calcium is most impor-
deficient to a degree that would cause serious illness. Folic tant in growing children for building bones. Folate deficien-
acid helps to prevent macrocytic anemia. Iodine is needed in cy leads to macrocytic anemia and is most likely to occur
small quantities for thyroid hormone synthesis. in adults with an inadequate diet, such as individuals with
PBD9 420, 442 BP9 302 PBD8 438, 670 BP8 306, 314 chronic alcoholism. Vitamin C deficiency occurs in individ-
uals who do not eat adequate amounts of fresh fruits and
vegetables.
49 F Humans do not generate vitamin C endogenously, so PBD9 443 BP9 303 PBD8 439, 659 BP8 304
they must have a continuous dietary supply. The lack of fresh
fruits and vegetables containing vitamin C led to scurvy in
many sailors and explorers in centuries past. Beriberi leads to 53 B This patient is morbidly obese. The extra weight puts
heart failure and results from thiamine deficiency. Kwashi- a strain on joints, particularly the knees, increasing the risk
orkor results from protein deficiency. Pellagra, characterized for osteoarthritis. Although the overall risk of cancer increases
by the “3 D's” of diarrhea, dermatitis, and dementia, is seen with obesity, the relationship between endometrial carcino-
in niacin deficiency. Pernicious anemia from vitamin B12 de- ma and obesity is well established. About 80% of individuals
ficiency can be complicated by neurologic deterioration in with type 2 diabetes mellitus are obese. The relationship of
severe cases. Rickets is seen in children who are deficient in diet and obesity to colon cancer is not as well established.
vitamin D. Worldwide, most hepatocellular carcinomas arise in indi-
PBD9 442–443 BP9 301–302 PBD8 437 BP8 312–313 viduals infected with hepatitis B; chronic alcoholism also is
a risk factor. Pulmonary adenocarcinoma is the least likely
CHAPTER 9 Environmental and Nutritional Diseases 129

bronchogenic cancer to be associated with smoking. Some 57 C IGF-1 increases in response to the hyperinsu-
renal cell carcinomas are associated with smoking. linemia of obese persons, who are also more likely to have
PBD9 444–448 BP9 302–305 PBD8 442 BP8 313–317 metabolic syndrome and type 2 diabetes mellitus. IGF-1
promotes cell growth as well as increased synthesis of
estrogens and androgens that favor neoplastic transforma-
54 E Leptin signaling from adipocytes that have taken up tion in cells. Adiponectin that is elaborated by adipocytes
an adequate supply of fatty acids ordinarily feeds back to acts as an insulin-sensitizing agent that prevents hyperin-
the hypothalamus, which decreases synthesis of neuropep- sulinemia. Aflatoxin from growth of Aspergillus on foods
tide Y. This neurotransmitter acts as an appetite stimulant, such as cereals acts as a carcinogen via mutation of the
and a decrease in its synthesis causes satiety. Adenosine is TP53 gene. Leptin normally decreases when fat stores are
a nucleoside used to treat cardiac dysrhythmias. Glucagon high, and derangements in its receptor may underlie some
opposes insulin by increasing hepatic glycogen storage. An forms of obesity. Selenium is a trace metal thought to have
increasing blood glucose level results in an increased release antioxidant properties protective against cancer. Trans fats
of insulin to promote glucose uptake into connective tissues, are derived from artificial hydrogenation of dietary fats
muscle, and adipose tissue. and are atherogenic by increasing LDL cholesterol while
PBD9 444–447 BP9 303–304 PBD8 439–441 BP8 314–317 decreasing HDL cholesterol.
PBD9 447–449 BP9 303–305 PBD8 442–444 BP8 313–316

55 C Appetite and satiety are controlled by a complex


system of short-acting and long-acting signals. The levels 58 D These findings suggest a diagnosis of diabetes mel-
of ghrelin produced in the stomach increase rapidly before litus. The patient is obese and most likely has type 2 diabetes
every meal and decrease promptly after the stomach is filled. mellitus. Type 1 and type 2 diabetes mellitus greatly increase
Leptin released from adipocytes exerts long-term control by the risk of early and accelerated atherosclerosis. Decreas-
activating catabolic circuits and by inhibiting anabolic cir- ing total caloric intake, particularly saturated fat, helps re-
cuits. α-MSH is an intermediate in leptin signaling. TRH and duce the risk of coronary artery disease. Vegetable and fish
CRF are among the efferent mediators of leptin signaling, oils are preferable to animal fat as sources of dietary lipid
and they increase energy consumption. for prevention of atherosclerosis. Reducing dietary sodium
PBD9 444–446 BP9 303–304 PBD8 441–442 BP8 314–315 helps to decrease blood pressure. Increased fluid intake aids
renal function. Dietary fiber helps to reduce the incidence of
diverticulosis. Vitamin A has no significant effect on athero-
56 B There are “obesity” genes that may play a role in genesis.
metabolic pathways. About 5% to 8% of obese adults have a PBD9 447 BP9 305 PBD8 442 BP8 316–317
mutation in the MC4R gene, and even though there are abun-
dant fat stores and plenty of leptin, the lack of MC4R to drive
energy consumption leads to weight gain. Leptin is the product 59 C More fruits and vegetables are recommended in the
of the ob gene, and mutations reduce leptin levels that sig- diet to help prevent colon cancer. Vitamins C and E have an
nal satiety, but such mutations are rare. Mutations in OB-R antioxidant and antimutagenic effect. Red wine in modera-
encoding the leptin receptor are seen in about 3% of cases of tion may have a beneficial antiatherogenic effect. Ice cream
early-onset obesity with hyperphagia in children. POMC is can include animal fat that may promote cancer, as would
in a catabolic pathway stimulated by leptin, but mutations the animal fat of beef. Chocolate includes vegetable fat,
are rare, and affected individuals typically have childhood which is not as harmful as animal fat, and persons consuming
onset with hyperphagia. Peroxisome proliferator–activated chocolate perform at a higher level.
receptor gamma (PPARγ) is stimulated by the thiazolidinedi- PBD9 448–449 BP9 306 PBD8 444 BP8 317–318
one drugs and leads to a reduction in free fatty acids, reduc-
tion in resistin, and decreased insulin resistance.
PBD9 444–447 BP9 303–305 PBD8 439–442 BP8 314–315
Chapter 8

Environmental and
Nutritional Pathology
QUESTIONS
Select the single best answer.

1 A 65-year-old woman with a history of smoking presents


with a 3-week history of chest pain and bloody sputum. An
X-ray film of the chest reveals a hilar lung mass. The surgical
specimen reveals a squamous cell carcinoma growing within
the lumen of a bronchus (shown in the image). Which of
the following chemical agents may be associated with the
pathogenesis of the cancer in this patient?

(A) Asbestos
(B) Beryllium
(C) Carbon
(D) Silica
(E) Tobacco

3 A 47-year-old man presents with a 6-week history of increas-


ing fatigue and dark-colored stools. Complete blood count
shows hemoglobin of 8.6 g/dL and microcytic, hypochro-
mic RBCs. Upper gastrointestinal endoscopy reveals a peptic
ulcer along the lesser curvature of the stomach. This patient’s
anemia is most likely caused by deficiency of which of the
following?
(A) Folic acid
(B) Iron
(A) Aflatoxins (C) Thiamine
(B) Aromatic hydrocarbons (D) Vitamin B12
(C) Benzene (E) Zinc
(D) Branched chain amino acids
(E) Carbohydrate polymers 4 A 49-year-old woman presents with a 2-month history of
yellow discoloration of her eyes, abdominal pain, weight loss,
2 A 75-year-old man who had worked in a shipyard for 25 years and low-grade fever (38.4°C, 101°F). Physical examination
dies of a thoracic tumor. Autopsy reveals a pleural tumor that shows a distended abdomen, right upper quadrant tenderness,
encases the lung. Interstitial pulmonary fibrosis and multiple and a palpable liver 2 cm below the costal margin. A liver
pleural plaques are noted. Numerous, brown, beaded ferrugi- biopsy reveals alcoholic hepatitis. The patient recovers and
nous bodies are also found in the lungs (shown in the image). is strongly advised to abstain from alcohol. However, she
Which of the following agents is most likely associated with subsequently imbibes some antifreeze containing ethylene
the pathogenesis of the cancer in this patient? glycol and develops acute failure of which organ?

73
74 Chapter 8

(A) Brain
10 A 12-year-old boy is rescued 2 days after becoming lost
(B) Heart in the Canadian woods in February. Physical examination
(C) Kidney shows he has gangrene of his fingers and toes. Which of the
(D) Liver following mechanisms of cell injury played the most impor-
(E) Pancreas tant role in mediating necrosis in the fingers and toes of this
patient?
5 A neonate was noted to have mild growth retardation and (A) Activation of proapoptotic proteins
facial dysmorphology. The mother was a known abuser of (B) Generation of activated oxygen species
several substances. This infant’s problem most likely resulted
(C) Lipid peroxidation
from maternal intake of which of the following?
(D) Membrane disruption by water crystals
(A) Alcohol
(E) Protein and DNA crosslinking
(B) Cocaine
(C) Ethylene glycol 11 A 35-year-old man is hospitalized with third-degree burns
(D) Heroin after being rescued from a house fire. Initially, he suffers from
(E) Marijuana shock and oliguria, but his renal function returns to normal
within a few days. Which of the following would be the most
6 A 10-year-old boy presents with irritability and ataxia. He likely cause of death if complications were to arise?
is subsequently found to have anemia, basophilic stippling (A) Ascites
of erythrocytes, and dark-gray pigmentation of the gums. (B) Curling ulcers with hemorrhage
Exposure to which of the following chemical agents is most (C) Cushing ulcers
likely associated with this disease?
(D) Pseudomembranous colitis
(A) Arsenic
(E) Sepsis
(B) Copper
(C) Lead 12 A healthy adult runs a marathon in the summer and develops
(D) Mercury hot dry skin, cessation of sweating, lactic acidosis, hypocal-
(E) Nickel cemia, and muscle necrosis (rhabdomyolysis). Which of the
following is the appropriate diagnosis?
7 A 50-year-old man presents for a routine physical examination, (A) Dysautonomia
which demonstrates an enlarged liver. During the visit, (B) Heat stroke
he describes memos from his supervisor at work regarding (C) Malignant hyperthermia
chronic exposure to vinyl chloride. The patient has an elevated
(D) Myotonic dystrophy
risk for which of the following tumors?
(E) Polymyositis
(A) Angiosarcoma of the liver
(B) Carcinoid tumor 13 A 26-year-old electrician is found unconscious in his backyard
(C) Hepatic adenoma beside a metal ladder and an exposed electrical wire, suffering
(D) Lymphoma from a deep burn on his right hand. Resuscitation attempts
(E) Metastatic colon cancer are unsuccessful. Which of the following was the most likely
cause of death?
8 A severely depressed 32-year-old man commits suicide (A) Cardiac arrhythmia
by running his car motor in his closed garage for several (B) Disseminated intravascular coagulation
hours. The mechanism of death in this case of carbon (C) Myocardial infarction
monoxide (CO) poisoning is through which of the following
(D) Rupture of the ascending aorta
mechanisms?
(E) Uncoupling of oxidative phosphorylation
(A) Displacement of oxygen on hemoglobin by CO
(B) Hepatocellular necrosis 14 A sailor on a nuclear-powered submarine is seen by a physi-
(C) Inhibition of protein synthesis cian after a breach in the reactor containment system. Physical
(D) Inhibition of the respiratory chain enzymes examination is unremarkable, but the patient subsequently
(E) Myocardial infarction develops profound pancytopenia. Hemoglobin is 7.8 g/dL,
WBC count is 900/µL, and platelets are 20,000/µL. How
9 A Japanese fisherman who lives in the vicinity of a plastics many rads of acute total-body radiation did this patient most
factory develops severe neurologic symptoms, including likely receive?
constriction of visual fields, paresthesias, ataxia, dysarthria, (A) 1
and hearing loss. Public health authorities find a number of (B) 3
similar cases in the local village. Exposure to which of the (C) 30
following chemical agents is most likely associated with the
(D) 300
pathogenesis of this man’s neurologic disease?
(E) 3,000
(A) Arsenic
(B) Copper 15 Another sailor on the submarine (see Question 14) dies
(C) Lead 10 days after the accident as a result of severe diarrhea and
(D) Mercury dehydration. Which of the following doses (in rads) of acute
(E) Nickel total-body radiation did this sailor most likely receive?
Environmental and Nutritional Pathology 75
(A) 1 neuropathy, difficulty balancing, and dementia. He dies sud-
(B) 3 denly of an arrhythmia, and at autopsy, lesions are found in
(C) 100 the mamillary bodies and the vicinity of the third ventricle.
(D) 300 The vitamin deficiency associated with these signs and symp-
(E) 1,000 toms is which of the following?
(A) Vitamin B1 (thiamine)
16 A 66-year-old woman presents with a 6-month history of scal- (B) Vitamin B12
ing and abnormal pigmentation of the skin. Her past medical (C) Vitamin D
history is significant for the treatment of thyroid cancer 1 year (D) Niacin
ago. Biopsy of lesional skin shows atrophy of the epidermis (E) Pyridoxine
and dense fibrosis of the dermis, which displays dilated super-
ficial blood vessels. These pathologic findings are most likely 22 A 40-year-old, malnourished woman presents with a 6-month
caused by previous exposure to which of the following? history of night blindness. Physical examination reveals kera-
(A) Chemotherapy tomalacia and corneal ulceration. Which of the following vita-
(B) Corticosteroids min deficiencies would be suspected in this patient?
(C) Organic iodine (A) Vitamin A
(D) Radiation therapy (B) Vitamin B2 (riboflavin)
(E) Tumor necrosis factor-α (C) Vitamin C
(D) Vitamin E
(E) Folic acid
17 A 28-year-old radar technician aboard an aircraft carrier has
been subjected to intense microwave radiation for 7 years. He
has an elevated risk for developing which of the following? 23 A 26-year-old woman presents to the emergency room with fever
and shaking chills. Her temperature is 38.7°C (103°F), pulse
(A) Aplastic anemia
120 per minute, and blood pressure 140/80 mm Hg. Physical
(B) Hodgkin disease
examination reveals a harsh systolic murmur. The patient devel-
(C) Lymphocytic leukemia ops a headache, slips into a coma and expires. The aortic valve is
(D) Myelogenous leukemia examined at autopsy (shown in the image). Which of the follow-
(E) None of the above ing is the most important risk factor for this pathologic finding?
18 A 15-year-old African boy has a history of tooth loss, gingivitis,
skin hemorrhages, multiple infections, and poor wound heal-
ing. Physical examination shows that the child is in the 20th
percentile for height and 10th percentile for weight. This patient
most likely has which of the following underlying conditions?
(A) Beri-beri
(B) Impetigo
(C) Kwashiorkor
(D) Pellagra
(E) Scurvy

19 A 24-year-old woman, who is a food faddist, eats only corn-


based foods. She presents with dermatitis, diarrhea, and
dementia. This patient most likely suffers from which of the
following conditions? (A) Alcoholism
(A) Beri-beri (B) Autoimmune disease
(B) Impetigo (C) Cigarette smoking
(C) Kwashiorkor (D) Intravenous drug abuse
(D) Marasmus (E) Oral contraceptive use
(E) Pellagra

20 A 45-year-old woman with longstanding Crohn disease and 24 A 50-year-old woman presents with easy fatigability, a smooth
severe fat malabsorption experiences a fracture of the femoral sore tongue, numbness and tingling of the feet, and weakness
neck after a minor contusion. This woman most likely has a of the legs. A complete blood count shows a megaloblastic
deficiency of which of the following vitamins? anemia that is not reversed by folate therapy. Hemoglobin is
(A) Vitamin B1 (thiamine) 5.6 g/dL, WBC count is 5,100/µL, and platelets are 240,000/
(B) Vitamin C µL. This patient most likely has a deficiency of which of the
following vitamins?
(C) Vitamin D
(A) Vitamin B1 (thiamine)
(D) Vitamin K
(B) Vitamin B2 (riboflavin)
(E) Niacin
(C) Vitamin B12
21 A homeless man, who is a chronic alcoholic, is brought to the (D) Vitamin K
hospital in a wasted state. He is noted to have a peripheral (E) Niacin
76 Chapter 8

sugar (160 mg/dL). An echocardiogram discloses significant


25 A starving, 4-year-old, African boy presents with apathy, gen-
cardiomegaly but no valvular abnormalities or evidence of old
eralized edema, and an enlarged fatty liver. The physician
myocardial infarcts. An angiogram shows normal coronary
notes that, despite generalized growth failure, subcutaneous
arteries. What is the most likely cause of his disease?
fat is preserved. What is the appropriate diagnosis?
(A) Chronic alcoholism
(A) Beri-beri
(B) Cigarette smoking
(B) Kwashiorkor
(C) Diabetes mellitus
(C) Marasmus
(D) Diffuse alveolar damage
(D) Pellagra
(E) Rheumatic heart disease
(E) Scurvy
31 A 28-year-old pregnant woman weighs 86.4 kg (190 lb) and
26 Petechial hemorrhages were noticed on the upper and lower gives birth after 39 weeks of gestation to a small baby weigh-
extremities of a 5-day-old infant. Hemorrhagic disease of the ing 2,300 g (5 lb, 1 oz). The baby has no congenital anoma-
neonate was most likely caused by a deficiency of which of the lies. Which of the following maternal factors in this case may
following vitamins? be associated with the infant’s low birth weight?
(A) Vitamin B2 (riboflavin) (A) Folic acid deficiency
(B) Vitamin D (B) Obesity
(C) Folic acid (C) Premature delivery
(D) Vitamin K (D) Previous oral contraceptive use
(E) Pyridoxine (E) Smoking

27 A 6-year-old girl is examined at a clinic in central Africa. 32 A 52-year-old man presents with sudden crushing chest pain
Physical examination reveals wasting of muscle and fat, and a and tachycardia. He admits to cigarette smoking, consump-
protuberant abdomen. Her pulse, blood pressure, and temper- tion of alcohol, and abuse of illicit drugs. An ECG is consistent
ature are low. The face appears wrinkled. There is no evidence with ischemic change in the anteroseptal region of the heart.
of generalized edema. What is the appropriate diagnosis? Laboratory studies show elevated serum levels of CK-MB and
(A) Beri-beri troponin I. Serum cholesterol is 240 mg/dL. Which of the fol-
lowing most likely contributed to this patient’s condition?
(B) Kwashiorkor
(A) Alcohol consumption
(C) Marasmus
(B) Cigarette smoking
(D) Pellagra
(C) Heroin addiction
(E) Scurvy
(D) Inadequate calcium intake
28 A 40-year-old, chronically ill man from a Vietnamese village (E) Marijuana use
presents with painful sores around his mouth. Physical exami-
nation reveals prominent fissures at the angles of his mouth. 33 A 52-year-old man presents with a chronic cough and shortness
Cheilosis in this patient is most likely caused by a deficiency of breath. He admits to smoking two packs of cigarettes a day
of which of the following vitamins? for 30 years. Pulmonary function tests reveal chronic obstruc-
(A) Vitamin A tive pulmonary disease. In counseling this patient, you advise
(B) Vitamin B1 (thiamine) him to stop smoking immediately. You also mention that, in
addition to emphysema, which of the following organs carries
(C) Vitamin B2 (riboflavin)
a significantly increased risk of smoking-related cancer?
(D) Vitamin B12
(A) Brain
(E) Vitamin C
(B) Liver
29 A 32-year-old woman is a vegan (i.e., a strict vegetarian who (C) Pancreas
eats no animal products of any kind). She is weak and pale, (D) Skin
and laboratory studies show a macrocytic, normochromic (E) Small bowel
anemia (hemoglobin = 6.2 g/dL). This patient most likely has
which of the following vitamin deficiencies? 34 A homeless, poorly nourished man collapses on the street and
(A) Vitamin A cannot be revived by the emergency medical technicians. A
(B) Vitamin B12 social service agency notes that he had a long history of abus-
ing many illicit drugs. An autopsy is performed and reveals a
(C) Vitamin D
moderately enlarged heart, with patent coronary arteries and
(D) Vitamin E
no valvular abnormalities. Microscopic examination shows
(E) Niacin patchy fibrosis of the myocardium. Which of the following
substances most likely caused this cardiomyopathy?
30 A 46-year-old man complains of weakness, dyspnea on exer-
(A) Cocaine
tion, and palpitations. His temperature is 37°C (98.6°F), pulse
95 per minute, respirations 24 per minute, and blood pres- (B) Heroin
sure 120/80 mm Hg. Physical examination reveals pulmonary (C) Lysergic acid diethylamide (LSD)
rales and peripheral edema. Laboratory studies show normal (D) Marijuana
serum cholesterol (180 mg/dL) and elevated fasting blood (E) Methamphetamine
Environmental and Nutritional Pathology 77

35 A 52-year-old, obese man (BMI = 34 kg/m2) presents to his 39 A 24-year-old beating victim is brought to the emergency room
physician with complaints of hoarseness for 2 months. He has with a 4-cm linear tear of the skin caused by blunt trauma.
worked in a chemical factory for 25 years and gives a his- Which of the following terms best describes this patient’s skin
tory of smoking, consumption of one or two beers a day, and lesion?
occasional use of illicit drugs. Physical examination reveals (A) Abrasion
enlarged and firm cervical lymph nodes. Direct laryngoscopy (B) Avulsion
reveals a fixed and enlarged left vocal cord, which appears (C) Blast injury
ulcerated. A biopsy of the lesion is interpreted by the patholo-
(D) Contusion
gist as squamous cell carcinoma. What is the most likely cause
(E) Laceration
of this man’s disease?
(A) Alcohol
(B) Benzene inhalation
(C) Cigarette smoking
ANSWERS
(D) Cocaine use 1 The answer is B: Aromatic hydrocarbons. These compounds
(E) High-fat diet (e.g., benzopyrene, methyl cholanthrene) are potent experi-
mental carcinogens. Aflatoxins (choice A) produce experi-
36 A 48-year-old man complains of weakness and easy fatigability mental liver cancer. The tumor in this patient is a squamous
for 6 weeks. He has worked for 20 years in a chemical factory cell carcinoma, which bears a strong resemblance to normal
that produces a variety of plastics and other synthetic com- squamous cells and synthesizes keratin, as evidenced by epi-
pounds. A complete blood count shows a hemoglobin level thelial pearls.
of 8.2 g/dL, WBC count of 45,000/µL, and a platelet count Diagnosis: Squamous cell carcinoma of the lung
of 40,000/µL. Examination of a bone marrow aspirate reveals
numerous malignant myeloblasts, and a diagnosis of acute
myeloid leukemia is made. Exposure to which of the follow- 2 The answer is A: Asbestos. Occupational exposure to
ing agents is the most likely cause of this patient’s hematologic asbestos poses a risk for the development of a pleural tumor
disease? termed malignant mesothelioma. Malignant disease may
become evident 20 to 30 years following exposure. The
(A) Benzene
inhalation of asbestos fibers also causes interstitial fibrosis of
(B) Benzopyrene
the lungs and pleural plaques consisting of dense connective
(C) Carbon tetrachloride tissue. Asbestos fibers coated with protein and iron are termed
(D) Glycerin asbestos (ferruginous) bodies. The other choices do not
(E) Trichloroethylene produce ferruginous bodies or cause mesothelioma.
Diagnosis: Mesothelioma, asbestosis
37 A 48-year-old woman complains she has had weakness, fatigue,
and easy bruisability for 2 months. She had worked as a tech- 3 The answer is B: Iron. Gastrointestinal hemorrhage leads to
nician in a nuclear energy plant for 15 years and was involved the loss of heme iron at a rate faster than it is replaced from
in an accident during which she was exposed to considerable dietary sources. The result is microcytic hypochromic anemia.
radiation. Physical examination reveals an enlarged liver and The anemias associated with deficiencies of folic acid (choice
spleen. What disease should you suspect as a likely cause of A) and vitamin B12 (choice D) are macrocytic.
her condition?
Diagnosis: Iron-deficiency anemia
(A) Chronic myelogenous leukemia
(B) Hairy cell leukemia 4 The answer is C: Kidney. The major toxicity of ethylene glycol
(C) Metastatic carcinoma of the breast is acute tubular necrosis of the kidney, which results in renal
(D) Metastatic carcinoma of the stomach failure. The compound has little effect on the other organs.
(E) Osteogenic sarcoma Diagnosis: Acute renal failure

38 A 16-year-old girl has suffered from severe celiac disease for 5 The answer is A: Alcohol. Fetal alcohol syndrome is the
years and reports continued steatorrhea. She suddenly devel- most common acquired cause of mental retardation in the
ops abdominal pain in the right lower quadrant. A complete United States. The common features of the syndrome include
blood count shows a hemoglobin level of 14 g/dL, WBC of intrauterine growth retardation, facial dysmorphology,
18,000/µL, with 84% neutrophils, and a platelet count of neurologic impairment, and other congenital anomalies. In
280,000/µL. A diagnosis of appendicitis is made, and tests cases with lesser manifestations, termed fetal alcohol effect,
before surgery reveal a prolonged prothrombin time of 17 children later suffer from mental retardation and minor
seconds (control = 2). What is the most likely cause of her dysmorphic features. Cocaine (choice B) may cause neonatal
coagulation problem? difficulties and heroin (choice D) intake may result in neonates
(A) Hemophilia A that are addicted to that opiate, but they are not associated
(B) Hemophilia B with characteristic facial dysmorphology.
(C) Hypolipidemia Diagnosis: Fetal alcohol syndrome
(D) Lymphoblastic leukemia
(E) Vitamin K deficiency 6 The answer is C: Lead. Chronic lead poisoning inhibits delta-
aminolevulinic acid dehydratase and ferrochelatase (enzymes
78 Chapter 8

essential for heme synthesis), thereby causing microcytic hyperthermia (choice C) occurs in surgical patients after anes-
hypochromic anemia. The inhibition of heme synthesis thesia.
leads to basophilic stippling of erythrocytes, which is due to Diagnosis: Heat stroke
residual ribosome clusters in the cytoplasm. Chronic exposure
of children to lead also leads to cognitive loss. Mercury 13 The answer is A: Cardiac arrhythmia. Electrical energy dis-
(choice D) poisoning has neurologic sequelae, but not these rupts the electrical system within the heart and frequently
hematologic characteristics. causes death through ventricular fibrillation. The force pro-
Diagnosis: Lead poisoning duced by high-voltage currents vaporizes tissue water and
produces extensive damage. The other choices are not conse-
7 The answer is A: Angiosarcoma of the liver. Occupational quences of powerful electrical currents. Although myocardial
exposure to vinyl chloride (used in the production of plastics) infarction (choice C) can cause an immediate arrhythmia, it
is associated with the development of this malignant tumor of reflects obstruction of the coronary circulation.
endothelial cells in the liver. Angiosarcoma is also associated Diagnosis: Electrical injury, cardiac arrhythmia
with exposure to arsenic (a component of pesticides) and
Thorotrast (a radioactive contrast medium used by radiologists 14 The answer is D: 300. Acute total-body irradiation of about
prior to 1950). None of the other tumors have been associated 300 rads causes depression of the bone marrow, and symp-
with occupational exposure to vinyl chloride. Hepatic adenoma toms related to granulocytopenia and thrombocytopenia
(choice C) is associated with the use of oral contraceptives. develop within 2 weeks. Anemia follows more slowly because
Diagnosis: Angiosarcoma of the liver red blood cells have a longer lifespan than leukocytes and
platelets. Exposure to 3,000 rads (choice E) is rapidly fatal
8 The answer is A: Displacement of oxygen on hemoglobin by CO. owing to central nervous system damage.
CO combines with hemoglobin with an affinity 240 times Diagnosis: Radiation sickness
greater than that of oxygen to form carboxyhemoglobin. At
concentrations above 50% carboxyhemoglobin, cerebral anoxia, 15 The answer is E: 1,000. A radiation dose of 1,000 rads
coma, convulsions, and death ensue. The other choices do not causes destruction of tissues composed of proliferating cells.
reflect the strong binding affinity of CO for hemoglobin. Damage to the gastrointestinal tract is the most serious con-
Diagnosis: Carbon monoxide poisoning sequence and ensues within days of exposure. Death results
from massive fluid loss from the denuded intestinal mucosa
9 The answer is D: Mercury. Mercury released into the and superimposed passage of bacteria through the damaged
environment may be bioconcentrated and enter the food intestine. The lower doses listed in the question do not destroy
chain, particularly predatory fish. Organic mercurials the gastrointestinal epithelium.
principally damage the brain, whereas inorganic mercury is Diagnosis: Radiation sickness
toxic to the kidneys. Large outbreaks attributed to methyl
mercury poisoning have been reported in Japan (fish) and 16 The answer is D: Radiation therapy. Ionizing radiation admin-
Iraq (fungicides). Poisonings by the other choices do not elicit istered for the treatment of cancer must first traverse the skin,
these neurologic symptoms. leading to radiation dermatitis. Skin biopsy shows atrophy of
Diagnosis: Mercury poisoning the epidermis and dense fibrosis of the dermis, which displays
dilated superficial blood vessels. In some cases, persistent
10 The answer is D: Membrane disruption by water crystals. ulcers require skin grafts. The other choices do not cause these
Exposure of the extremities to severe cold results in the crys- dermal findings.
tallization of tissue water, which causes cellular disruption and
Diagnosis: Radiation injury
vascular changes, resulting in frostbite. Localized thrombosis
often leads to focal ischemia and gangrene of toes and fingers.
17 The answer is E: None of the above. The absorption of micro-
Mechanical disruption of cellular membranes by ice crystals
wave energy produces only heat and is not associated with any
occurs during both freezing and thawing. The other choices
known health risks.
are associated with cell death, but they are not early events in
frostbite-induced necrosis. Diagnosis: Radiation injury
Diagnosis: Frostbite
18 The answer is E: Scurvy. Vitamin C is essential for collagen syn-
11 The answer is E: Sepsis. The most common cause of death thesis, and its deficiency results in poor wound healing. Perifol-
in seriously burned patients is sepsis after infection of the licular hemorrhages arise from capillaries that have weak walls
burned skin. Gastric ulcers (stress or Curling ulcers, choice and are easily damaged by minor trauma. Impaired collagen
B) are occasionally encountered in burn patients, but they do synthesis leads to gingivitis and alveolar bone resorption, result-
not represent a common cause of death. Cushing ulcers of the ing in loss of teeth. Wound healing requires collagen synthesis
stomach (choice C) are associated with trauma to the central and is impaired in patients with vitamin C deficiency (scurvy).
nervous system. Diagnosis: Scurvy, vitamin C deficiency
Diagnosis: Sepsis, thermal injury
19 The answer is E: Pellagra. Niacin deficiency leads to the 3
12 The answer is B: Heat stroke. Exertional heat stroke occurs “Ds”: dermatitis, diarrhea, and dementia. A swollen, fissured
in healthy men during unusually vigorous exercise, particu- tongue and chronic watery diarrhea are also characteristic.
larly when the ambient temperature is high. Lactic acidosis, Dementia reflects degeneration of ganglion cells in the cere-
hypocalcemia, and rhabdomyolysis may be severe problems. bral cortex.
Myoglobinuric acute renal failure is not uncommon. Malignant Diagnosis: Pellagra, niacin deficiency
Environmental and Nutritional Pathology 79
is the rule. These changes are reversible if and when sufficient
20 The answer is C: Vitamin D. Lipid malabsorption inter-
protein is made available.
feres with the absorption of vitamin D, thereby leading to
Diagnosis: Kwashiorkor
a deficiency state. In adults, vitamin D deficiency results in
osteomalacia, a disorder characterized by inadequate mineral-
26 The answer is D: Vitamin K. Hemorrhagic disease of the new-
ization of newly formed bone matrix. The consequent weak-
born may be caused by a deficiency of vitamin K. Vitamin
ness of bone is associated with a vulnerability to spontaneous
K is an important coagulation factor, which is necessary for
fractures. Vitamin D deficiency in children is termed rickets.
the carboxylation and activation of prothrombin, as well as
Deficiencies in the other choices are not associated with these
of clotting factors VII, IX, and X. Newborn infants frequently
bone abnormalities.
exhibit vitamin K deficiency because the vitamin is not trans-
Diagnosis: Osteomalacia, vitamin D deficiency
ported well across the placenta, and the sterile gut of the new-
born does not have bacteria to produce it. The other vitamin
21 The answer is A: Vitamin B1 (thiamine). This man suffers
deficiencies do not impair coagulation.
from beri-beri (heart) and Wernicke-Korsakoff syndrome
Diagnosis: Vitamin K deficiency, hemolytic disease of the
(brain). Thiamine deficiency in deteriorated alcoholics results
newborn
in encephalopathy, peripheral neuropathy, and other disorders.
Atrophy of the mammillary bodies, with loss of ganglion cells
27 The answer is C: Marasmus. Deficiency of all elements of the
and rupture of small blood vessels, is characteristic. Deficien-
diet leads to marasmus. The condition is common throughout
cies of the other vitamins are not related to these disorders.
the nonindustrialized world, particularly when breast feeding
Diagnosis: Beri-beri, Wernicke encephalopathy
is stopped, and a child must subsist on a calorically inadequate
diet. The pathological changes are similar to those in starving
22 The answer is A: Vitamin A. Vitamin A deficiency causes adults and consist of decreased body weight, diminished sub-
squamous metaplasia at a number of sites. In the cornea, it cutaneous fat, a protuberant abdomen, muscle wasting, and a
leads to xerophthalmia (dry eye), which may progress to soft- wrinkled face. In general, the child appears as a “shrunken old
ening of the tissue (keratomalacia) and corneal ulceration. person.” Wasting and increased lipofuscin pigment are seen
Deficiencies of the other vitamins are not related to these clini- in most visceral organs, especially the heart and the liver. No
cal and pathologic findings. edema is present. The pulse, blood pressure, and temperature
Diagnosis: Vitamin A deficiency, keratomalacia are low, and diarrhea is common. Because immune responses
are impaired, the child suffers from numerous infections. An
23 The answer is D: Intravenous drug abuse. The introduction of important consequence of marasmus is growth failure. If these
bacteria by intravenous drug abuse may lead to septic compli- children are not provided with an adequate diet during child-
cations in many organs. Bacterial endocarditis, often involving hood, they will not reach their full potential stature as adults.
Staphylococcus aureus, may occur on both sides of the heart. Kwashiorkor (choice B) results from deficiency of protein in
Infected emboli can occlude vessels leading to gangrene. the diet. Choices A, D, and E result from deficiencies of vita-
Infected emboli in the brain can cause cerebral abscess. The min B1, nicacin, and vitamin C, respectively.
photograph shows adherent vegetations on the aortic valve. Diagnosis: Marasmus
These vegetations are composed of platelets, fibrin, cell debris,
and masses of organisms. In addition to intravenous drug 28 The answer is C: Vitamin B2 (riboflavin). Cheilosis refers to
abuse, risk factors for bacterial endocarditis include congeni- fissures at the angles of the mouth and is a common finding
tal heart disease (children), rheumatic heart disease, prosthetic in patients with vitamin B2 (riboflavin) deficiency. Riboflavin
heart valves, transient bacteremia, and diabetes. Although cer- participates in the synthesis of flavin mononucleotides. Sebor-
tain autoimmune diseases (choice B) are associated endocardi- rheic keratosis and interstitial keratitis of the cornea also occur
tis (e.g., Libman-Sacks), the verrucous vegetations are sterile in patients with vitamin B2 (riboflavin) deficiency. Except for
(nonbacterial). The other choices are not associated with a vitamin A deficiency (which does not cause cheilosis), the
significantly increased risk of bacterial endocarditis. other choices (B, D, and E) do not affect the skin.
Diagnosis: Bacterial endocarditis Diagnosis: Cheilosis, vitamin B2 (riboflavin) deficiency

24 The answer is C: Vitamin B12. Except for a few rare situations, 29 The answer is B: Vitamin B12. Vitamin B12, which is neces-
vitamin B12 (cyanocobalamin) deficiency is usually a result of sary for DNA synthesis, is contained only in animal products,
pernicious anemia, an autoimmune disease of the stomach. including eggs. Extreme vegetarians may suffer vitamin B12
Vitamin B12 is required for DNA synthesis, and its deficiency deficiency after many years of a restricted diet. The result is a
results in large (megaloblastic) nuclei. macrocytic anemia similar to that seen in pernicious anemia.
Diagnosis: Vitamin B12 deficiency, pernicious anemia Macrocytic anemia is not a consequence of the other choices.
Diagnosis: Vitamin B12 deficiency, megaloblastic anemia
25 The answer is B: Kwashiorkor. Kwashiorkor is a syndrome
that results from a deficiency of protein in a diet relatively 30 The answer is A: Chronic alcoholism. Alcoholic cardiomyopa-
high in carbohydrates. It is one of the most common diseases thy correlates with the total lifetime dose of alcohol and leads
of infancy and childhood in the nonindustrialized world. It to dilation and hypertrophy of the heart. As in this case, the
usually occurs after an infant is weaned, when a protein-poor disorder may cause congestive heart failure. Cigarette smoking
diet, consisting principally of staple carbohydrates, replaces (choice B) and diabetes mellitus (choice C) are associated with
mother’s milk. Unlike marasmus (choice C), the disorder fea- coronary artery disease, and rheumatic heart disease (choice
tures edema, large fatty liver, and depigmentation of the skin. E) features valvular abnormalities.
Extreme apathy is notable, diarrhea is common, and anemia Diagnosis: Alcoholic cardiomyopathy
80 Chapter 8

from smoking (>80% of cases). In some large studies, all


31 The answer is E: Smoking. Fetal tobacco syndrome refers to
deaths from cancer of the larynx occurred in smokers. There
the deleterious effects of maternal cigarette smoking on the
is no epidemiologic evidence that the other choices are risk
development of the fetus. Infants born to women who smoke
factors for laryngeal cancer, although chronic alcoholism may
during pregnancy are, on average, 200 g lighter than infants
be associated with a slightly increased risk.
born to women who do not smoke. These infants are not born
preterm, but rather, are small for gestational age. The noxious Diagnosis: Laryngeal carcinoma
effect of smoking on the fetus is mirrored by its effect on the
36 The answer is A: Benzene. Virtually all cases of acute and
uteroplacental unit. The incidences of abruptio placentae, pla-
chronic benzene toxicity have occurred against the back-
centa previa, uterine bleeding, and premature rupture of the
ground of industrial exposure. Acute benzene poisoning
membranes are all increased in women who smoke. Evidence
primarily affects the central nervous system, and death results
indicates that the injurious effects of maternal cigarette smoking
from respiratory failure. However, the long-term effects of
are not limited to the fetus but extend to the physical, cognitive,
benzene exposure have attracted the most attention. The bone
and emotional development of children at older ages. Choices
marrow is the principal target in chronic benzene intoxication.
A, B, and D are not associated with low birth weight infants.
Patients who develop hematologic abnormalities characteristi-
Diagnosis: Fetal tobacco syndrome
cally exhibit hypoplasia or aplasia of the bone marrow and
pancytopenia. Aplastic anemia usually is seen while the work-
32 The answer is B: Cigarette smoking. Cigarette smoking is
ers are still exposed to high concentrations of benzene. In a
recognized as a major independent risk factor for myocardial
substantial proportion of cases of benzene-induced anemias,
infarction and acts synergistically with other risk factors such
acute myeloblastic leukemia develops. Overall, the risk of leu-
as high blood pressure and elevated blood cholesterol levels.
kemia is increased 60-fold in workers exposed to the highest
It not only serves to precipitate initial myocardial infarction
atmospheric concentrations of benzene. The other choices are
but also increases the risk for second heart attacks and dimin-
not linked to the development of leukemia.
ishes survival after a heart attack among those who continue
to smoke. Smoking also increases the incidence of sudden Diagnosis: Acute myelogenous leukemia
cardiac death, possibly by exacerbating regional ischemia.
37 The answer is A: Chronic myelogenous leukemia. The evi-
Atherosclerosis of the coronary arteries and the aorta is more
dence that whole-body radiation can lead to cancer is incon-
severe and extensive among cigarette smokers than among
trovertible and comes from animal experiments and studies
nonsmokers, and the effect is dose related. Chronic alcohol
of the effects of occupational exposure, radiation therapy for
consumption (choice A) actually protects against coronary
nonneoplastic conditions, the diagnostic use of certain radio-
artery disease, although dilated cardiomyopathy may develop.
isotopes, and the atom bomb explosions. Some survivors of
The other choices are not related to heart disease.
the atom bomb explosions and patients subjected to spinal
Diagnosis: Myocardial infarction
radiation later developed chronic myelogenous leukemia.
Although the other choices may lead to hepatosplenomegaly,
33 The answer is C: Pancreas. Cancer of the pancreas has shown
they are not linked to acute radiation exposure.
a steady increase in incidence, which is partly related to ciga-
rette smoking. The risk for adenocarcinoma of the pancreas in Diagnosis: Chronic myelogenous leukemia
male smokers is elevated two- to threefold, and a clear dose- 38 The answer is E: Vitamin K deficiency. Vitamin K deficiency is
response relationship exists. In fact, men who smoke more common in severe fat malabsorption, as seen in celiac sprue and
than two packs a day have a five times greater risk of devel- biliary tract obstruction. The destruction of intestinal flora by
oping pancreatic cancer than nonsmokers. Smoking does not antibiotics may also result in vitamin K deficiency. Vitamin K,
increase the risk of cancer for the other choices. which confers calcium-binding properties to certain proteins, is
Diagnosis: Chronic obstructive pulmonary disease important for the activity of four clotting factors: prothrombin,
factor VII, factor IX, and factor X. Deficiency of vitamin K can
34 The answer is A: Cocaine. Cocaine overdose leads to anxiety, be serious because it can lead to catastrophic bleeding.
delirium, and occasionally seizures. Cardiac arrhythmias and
Diagnosis: Vitamin K deficiency
other effects on the heart may cause sudden death in other-
wise healthy persons. Chronic abuse of cocaine is associated 39 The answer is E: Laceration. A laceration is a linear tear of the
with the occasional development of a characteristic dilated skin produced by a force that causes unidirectional displace-
cardiomyopathy, probably because of its effects in small, ment. A surgical incision is a controlled laceration. Internal
intramyocardial, coronary arteries. The other choices do not organs may also be lacerated by trauma or by the surgeon.
cause cardiomegaly. An abrasion (choice A) is a skin defect caused by crushes or
Diagnosis: Cocaine cardiomyopathy scrapes. Avulsion (choice B) is a tearing away or forcible sepa-
ration. A contusion (choice D) is a localized mechanical injury
35 The answer is C: Cigarette smoking. Cancers of the lip, with focal hemorrhage.
tongue, and buccal mucosa occur principally (>90%) in Diagnosis: Laceration
tobacco users. Cancer of the larynx and esophagus also result

You might also like